Что будет если перепутать полярность на аккумуляторе при установке: Что будет если перепутать клеммы аккумулятора. Установленный на машине, а также при зарядке

Содержание

Что будет, если перепутал клеммы на аккумуляторе

С новой аккумуляторной батареей неприятности случаются достаточно редко, особенно если она необслуживаемая. Поэтому и необходимость в её снятии возникает нечасто. И всё же даже такие АКБ нуждаются в периодической зарядке, и эта операция требует внимательности во время подключения зарядного устройства. Если с установкой батареи на автомобиль обычно проблем не бывает, то при зарядке вероятность перепутать провода не такая уж маленькая. Насколько опасна такая ошибка, и что будет, если допустить её во время прикуривания от другого аккумулятора?

Перепутал клеммы аккумулятора: возможные последствия

Самый простой ответ на вопрос о том, что будет, если перепутать клеммы автомобильного аккумулятора, короток: не ждите ничего хорошего. При самом оптимистичном сценарии вам придётся менять плавкие предохранители, но зачастую последствия гораздо печальнее. Перечислим их.

Замыкание

Первое, что нужно уяснить: переполюсовка (это не совсем правильный термин, но суть проблемы отражает верно) – это короткое замыкание. И оно всегда сопровождается визуальными и звуковыми эффектами, заметить которые невозможно: искрами, характерными щелчками, не исключена и задымленность.

Если у вас очень хорошая реакция, всех нижеперечисленных проблем можно избежать, просто отдёрнув руку с проводом. Увы, так бывает не всегда. А значит, читаем дальше.

Воспламенение, пожар

Самый катастрофический вариант бывает, если водитель «впал в ступор» после того, как после неправильного подключения с клемм начался сыпаться град искр. Переполюсовка – это, по существу, КЗ, при котором провода начинают раскаляться, плавя оплётку. Изоляция не всегда бывает негорючей, а если в месте замыкания проводка проходит вплотную к другим горючим материалам или контактирует с техническими жидкостями, риск возгорания увеличивается кратно.

В таких случаях действовать нужно быстро, не допуская длительного замыкания при неправильном соединении проводов. Если это провода от зарядки, можно выдернуть вилку ЗУ из розетки – это будет безопаснее, чем сбрасывать клеммы.

Падение мощности АКБ

Если вы перепутали клеммы аккумулятора не на машине, а когда ставили батарею на зарядку, то проблемы будут локализованы только самой батареей и зарядным устройством.

Если аккумулятор был полностью разряжен, то его подключение с перепутанными проводами приведёт к зарядке, при которой его клеммы сменят полярность – плюсовая станет минусовой и наоборот. Вот эта процедура и называется переполюсовкой. В некоторых случаях её используют целенаправленно, но падения ёмкости батареи при этом не миновать.

Другое дело – ошибка с проводами в отношении частично заряженной АКБ. Здесь, скорее всего, пострадает зарядное устройство, причём безвозвратно. Вследствие короткого замыкания.

Выход из строя ЭБУ

Если пожар автомобиля следует считать катастрофическим сценарием, поскольку в этом случае вы лишаетесь транспортного средства, то перегорание бортового компьютера – вторая по последствиям неприятность. Без электронного бока управления современный легковой автомобиль работать не может, а сам блок – вещь дорогостоящая. Потеря настроек – ещё одно последствие перегорания электронного блока БК.

Перегоревшие предохранители

Это самый оптимистичный вариант (по деньгам, разумеется). Но, если честно, надеяться на то, что всё обойдётся только заменой плавких предохранителей, не стоит. Даже если у вас неплохая реакция, до того, как от возрастания тока перегорит рассчитанный на определённое сопротивление проводок, может пострадать чуткая к перепадам напряжения электроника.

Впрочем, и с перегоревшими предохранителями могут возникнуть проблемы, поскольку далеко не всегда удаётся сразу определить, что именно не работает. Так что при форс-мажоре, вызванном вашей ошибкой с подключением аккумулятора, не поленитесь перед поездкой проверить работоспособность всех потребителей.

Поломка генератора

Если вы перепутали клеммы АКБ, первым, кто про это узнает, будет именно генератор. Правда, он хорошо защищён против таких ошибок наличием диодного моста, но встречаются модели (в основном устаревшие или очень дешёвые), где он отсутствует, и тогда генератор первым подвергается воздействия короткого замыкания и перегорает.

Отказ работы сигнализации

Поскольку сигнализация обычно подключается непосредственно к батарее, вероятность того, что она сгорит, достаточно велика. Особенно если её конструкция предусматривает наличие триггеров – они изменения полярности не переносят.

Частичная оплавка проводов

Об этой проблеме мы уже упоминали – вследствие КЗ через провода начинает течь ток, превышающий номинал допустимого значения. Сопротивление растёт, металл быстро греется, изоляция плавится. Так что допустили ошибку – обязательно проверьте целостность проводов.

Переплюсовка

Инженеры сделали все, чтобы автомобилисты не могли перепутать клеммы аккумулятора. Они предусмотрели разные крепления и делают провода короткими. Однако это все равно не защищает от путаницы.


Поможет ли в холода установка батареи большей емкости на авто? Подробнее

Начнем с самого частого случая, когда меняют местами зажимы зарядного устройства. Этот прибор не имеет клемм с разными размерами, у них они быстросъемные, и перепутать их легко. Устройство, как правило, отреагирует на это перегоранием предохранителя.

Самодельные зарядные устройства такой защиты не имеют и будут подавать питание на батарею даже при смене полярности. Тогда в АКБ происходит процесс, называемый «переплюсовка». В итоге наносится вред аккумулятору и уменьшается его срок службы.

Раньше, когда аккумуляторы были большими и надежными, они выдерживали такую процедуру легко, а сейчас с уменьшением толщины пластин «переплюсовка» рискует закончиться для них фатально. Страдают пластины, и разрушаются ячейки, что приводит к микрозамыканиям внутри батареи. Если неправильная установка зарядного устройства быстро обнаруживается, то батарею можно спасти. Необходимо полностью разрядить ее с помощью источника тока, к примеру автомобильной лампочки, а после чего уже можно заряжать батарею с прежней полярностью. В этом случае аккумулятор еще послужит. Хотя его емкость и снизится.

Если перепутал клеммы при зарядке автомобильного аккумулятора

Обычно именно так и случается. Дело в том, что при установке АКБ на штатное место неправильно подключить провода маловероятно, даже если вы перепутаете полярность – плюсовая и минусовая клеммы имеют разный диаметр.

Другое дело – подключение ЗУ к батарее. Крокодилы по размерам не отличаются, а вероятность перепутать провода хоть и небольшая, но существует.

Если вы достаточно быстро сообразили, что ошиблись, последствий можно избежать. В противном случае есть риск, что сгорит зарядное устройство. В случае глубокой разрядки аккумулятора неправильное подключение грозит переполюсовкой, то есть сменой полярности клеммных выводов. Если конструкция АКБ позволяет использовать её в таком виде – можете попробовать, но о переполюсовке нужно всегда помнить.

Лучше всё же вернуть нормальную полярность батареи, полностью разрядив её (обязательно медленно!) и зарядив уже с нормальной полярностью подключения проводов. Разумеется, самое опасное в этом случае – глубокий разряд. Для некоторых разновидностей аккумуляторов (например, кальциевых), такая процедура крайне нежелательна.

Использование самодельных зарядок допустимо, если они оснащены плавким предохранителем, который защитит от перегорания ЗУ. Заводские устройства в своём большинстве такую защиту имеют.

Впрочем, это не относится к дешёвым китайским зарядным устройствам. Если вы перепутали клеммы АКБ при зарядке, они, скорее всего, такого надругательства не выдержат.

Что будет, если перепутать

Вариантов развития сценарий несколько. В любом случае, ничего хорошего из этого не произойдет.

1. Самое безобидное – это короткое замыкание,

при котором перегорят предохранители. Если все случится именно так, то можно считать, что вам крупно повезло: именно предохранители спасут все остальные элементы цепи.

2. Сам АКБ тоже может не выдержать замыкания

, особенно актуально это для уже поживших батарей.

3. КЗ может стать в свою очередь причиной возгорания:

цепь состоит из множества мельчайших проводков. Загоревшаяся оплетка может за мгновения спровоцировать пожар, ведь рядом так много воспламеняющихся веществ.

4. Если не сработают предохранители, то не сладко придется ЭБУ.

Компьютер просто парализует, в лучшем случае. В худшем – он может вовсе выйти из строя, а замена «мозгов» — это совершенно не дешевое удовольствие.

5. Замыкание нередко выводит из строя диодный мост генератора.

Опять же, если ситуацию не спасут предохранители.

6. Любой электроприбор сети может пострадать:

магнитола, когндиционер, сигнализация и т.д.

Была полезной статья? Поставьте класси поделитесь в соц. сетях !Подписывайтесь на канал, если любите автомобили!

Если перепутал клеммы аккумулятора при прикуривании

Такая операция, как прикуривание «дохлого» аккумулятора от заряженного, достаточно редкая. Но она таит в себе немало опасностей и для донора, и для акцептора.

Дело в том, что при последовательном и неправильном подключении проводов мы будем иметь дело с коротким замыканием с номиналом 24 В, а не 12. Если вы будете использовать некачественные провода малого сечения, они, скорее всего, просто перегорят в месте, где имеется самое значительное снижение их диаметра. С повышением сечения вероятность перегорания проводов снижается. По крайней мере, этому будет предшествовать процесс их быстрого нагрева, но здесь уже возрастает вероятность возникновения пожара.

Так что будьте осторожными при подключении проводов, особенно не заводских (последние маркируются цветами). Важно, чтобы положительная клемма донорского АКБ была соединена с такой же плюсовой клеммой на разряженной батарее, причём этот провод подключается первым (сначала крокодил цепляется на авто, которое необходимо завести).

После успешного прикуривания не менее важно соблюсти правильную очерёдность отключения проводов: сначала минусовый, и только после этого – положительный.

Помните, что к прикуриванию следует прибегать, только если другого выхода нет – эта процедура очень опасна для бортовой электроники из-за неизбежных скачков напряжения.

НЕПРАВИЛЬНОЕ ПРИКУРИВАНИЕ

Если водитель перепутал клеммы аккумулятора при прикуривании, то происходит следующее: мы получаем две последовательно соединенных батареи, выходы которых замкнуты накоротко. Получаем короткое замыкание уже под напряжением в 24 вольта. Подавляющее большинство продающихся в магазинах комплектов проводов, у которых сечение жил и так недостаточно, при этом моментально сгорит, оставив на руках незадачливого водителя ожоги. Если же провода с достаточным сечением, то перед перегоранием они успеют нагреться.

Поэтому всегда проверяйте правильность подсоединения проводов. Если у заводских комплектов хотя бы есть цветовая маркировка, то самодельные провода чаще нарезаются из одного куска и по цветам не отличаются. Пометить их можно цветной изолентой на «крокодилах» — это снизит риск неправильного подключения.

Первыми соединяются «плюсовые» клеммы – сначала «крокодил» крепится на прикуриваемом аккумуляторе, затем на прикуривающем. Масса подключается в обратном направлении – сначала на прикуривающем аккумуляторе. Таким образом цепь замыкается на последнем шаге на прикуриваемом автомобиле.

Так как правильно отсоединять клеммы аккумулятора не менее важно, порядок отключения тоже строго определен. Первой отключается масса, после этого любые манипуляции с проводами не будут давать риск короткого замыкания

Помните, что прикуривание для бортовой электроники – это нештатная ситуация, создающая повышенную нагрузку на генератор, чрезмерные скачки напряжения в сети прикуриваемого автомобиля во время вращения стартера.

Источник

Обычное ЗУ: заряд и никакого контроля


Такие часто идут в комплекте с аккумуляторами. И для тех АКБ, с которыми они продаются, они подходят идеально. Но с остальными батареями могут возникнуть проблемы. Что с такими зарядными устройствами не так?

  • Работают по жесткому циклу. Все параметры зарядки фиксированы, вы не можете менять ни силу тока, ни продолжительность зарядки. Если у вас древнее устройство, купленное в комплекте с АКБ 800 мАч, современные аккумуляторы емкостью 2400 мАч вы будете заряжать ну очень долго.
  • Никак не контролируют заряд. Вам может показаться, что контроль все-таки есть, ведь по окончании зарядки загорается зеленый светодиод. Но чаще всего он загорается… по таймеру. То есть, например, производитель рассчитал, что с установленной силой тока элементы емкостью 2400 мАч будут заряжены через 8 часов — значит, через 8 часов можно заканчивать зарядку и сигнализировать о готовности.
  • Не имеют «защиты от дурака». Если вы перепутали полярность элементов при установке в ЗУ, это останется на вашей совести.
  • Заряжают АКБ только пАрами. Да, один зарядить у вас скорее всего не получится.

Неправильное соединение клемм при прикуривании.

При прикуривании ситуация ухудшается в разы: бодрый мощный источник + АКБ, что хотят «подкурить» (еще не совсем дохлая) + нередко заведенный двигатель = адская смесь, практически фейерверк под капотами и неминуемое возгорание, если не осуществит отсоединение проводов в течении 1-2 минут (о том, как правильно прикурить смотрите здесь).

Но ведь провода путают не только на автомобиле, но и дома, при зарядке, что происходит тогда.

По большому счету в таких случаях ничего не происходит. Аккумулятор всегда остается цел, единственное, что поддается угрозе, – это зарядник. Причем и он, как правило, всегда остается жив, сгорает лишь его предохранитель, который легко меняется.

Последствия неправильного подключения аккумулятора

Сразу развеем миф о том, что «задом наперед» подключенный аккумулятор запускает двигатель в обратную сторону. Начнем со стартера: если статор с обмоткой, а не с постоянными магнитами, то при переполюсовке направление тока изменится одновременно и в статоре, и в роторе, стартер начнет вращаться в ту же сторону, что и раньше. Вспомните о коллекторных двигателях электродрелей, болгарок и другого инструмента – они работают на переменном токе, направление которого меняется постоянно.

Стартер с постоянными магнитами на статоре действительно начнет вращаться в другом направлении, но двигатель провернуть не сможет. Бендикс не даст передать усилие в «неправильную» сторону, для того он и предназначен.

Даже если бы Вы и умудрились раскрутить мотор в обратном направлении, четырехтактный мотор при этом начинает забирать воздух из выпускного коллектора, а выбрасывать отработавшие газы во впускной. Так что ни карбюраторный, ни инжекторный двигатель так не завести, разве что дизель с механическим ТНВД запустится.

Если водитель перепутал полярность аккумулятора при установке, то одним из самых серьезных последствий будет то, что однозначно пострадает генератор. Посмотрите на схему его диодного моста (на примере простейшей «жигулевской» схемы):

Полупроводниковый диод – это элемент с односторонней проводимостью, не зря его символическое изображение похоже на стрелку. В направлении этой «стрелки» он проводит ток, в обратном – нет. Когда аккумулятор подключен правильно, то, пока генератор не работает (напряжение на его обмотках меньше, чем напряжение аккумулятора), диоды заперты – для аккумулятора генератор нагрузка, потребляющая несколько миллиампер из-за наличия в диодах небольшого обратного тока.

Если перепутать клеммы на аккумуляторе, то диоды в мосте откроются, и аккумулятор окажется замкнут через них. Это приведет к перегреву диодного моста и выходу диодов из строя с характерным запахом и хлопком из-за разрушения их корпусов.

Cгоревший диодный мост генератора

А вот для бортовой электроники переполюсовка вредна не всегда. Любое электронное устройство, если его не собирали на уроках труда китайские дети, имеет защиту от переполюсовки. Реализуется она просто с помощью тех же диодов: тут они открыты при «правильном» подключении, а при «неправильном» отрезают нагрузку от бортовой сети. Однако диоды по цепи питания редко используются в мощных потребителях тока – магнитолах и усилителях. А в них есть крайне уязвимые для переполюсовки элементы – электролитические конденсаторы, устанавливаемые для сглаживания пульсаций напряжения как раз в цепях питания.

Электролитические конденсаторы имеют высокую удельную емкость, поэтому популярны в электронике. Но их слабое место – это работа только при определенной полярности напряжения на обкладках, при переполюсовке происходит нагрев, обильное газовыделение (конденсатор вздувается), затем – взрыв. Чем больше емкость и габариты конденсатора, тем все это сильнее – если конденсатор на несколько микрофарад слегка щелкнет, то «банки» на тысячи микрофарад, применяемые в усилителях, взрываются.

Вышедшие из строя конденсаторы

И множество реле в проводке может издать тот же звук, что и диоды в мосте генератора. Дело в том, что в обмотках электромагнитных реле при отключении напряжения возникает резкий выброс напряжения, имеющего обратную полярность относительно источника питания. Для устранения вредного влияния таких импульсов на работу чувствительной электроники принято использовать диоды, подключаемые в обратном направлении. Пока на обмотку подано напряжение из бортовой сети, диод заперт, а при размыкании обмотки он открывается импульсом индуктивного выброса и гасит его. При переполюсовке аккумулятора такие диоды перегорят.

Прямая и обратная полярность аккумулятора

Любой водитель должен знать о своем автомобиле все. Это необходимо для того, чтобы поддерживать работоспособность своего железного коня. Однако далеко не все знают, что такое полярность аккумулятора, из-за чего могут подключить его неправильно и создать тем самым для себя кучу проблем.

Полярность — это расположение внешних токовыводов, которые находятся на верхней или лицевой крышке аккумулятора. Есть 2 самых популярных схемы их расположения:

  • обратная;
  • прямая.

Есть и другие варианты расположения, но они в большей степени применяются в азиатских странах. Другими словами, это расположение клемм и исходя из того, с какой стороны расположена плюсовая клемма и характеризует АКБ.

Содержание статьи

  • Что значит прямая и обратная?
  • Как определить
    • Отличия
  • Какие будут последствия, если перепутал клеммы?
    • При установке на автомобиль
  • Полезное видео
    • При зарядке аккумулятора
  • Заключение

Что значит прямая и обратная?

Рассмотрим более подробно каждый вид аккумуляторов:

  • С прямой полярностью. Такая разработка актуальна в основном для отечественных инженеров. Ее особенность заключается в том, то вывод на плюс «+» располагается с левой стороны, а на минус «-» — с правой стороны верхней крышки корпуса.
  • С обратной полярностью. Такие батареи в основном применяются в странах Европы. Полярность располагается следующим образом: минус «-» находится слева, а плюс «+» — справа.

Справедливости ради, стоит отметить, что далеко не все автомобили из Европы имеют батареи с обратной полярностью, зачастую те модели, которые собираются в странах СНГ, комплектуются с прямой полярностью.

Как определить

Определение полярности не составляет никаких сложностей. Необходимо поставить перед собой АКБ таким образом, чтобы наклейка была повернута к вам. Теперь необходимо просто посмотреть, с какой стороны находится плюс «+». Если плюс расположен справа – полярность обратная, если слева – прямая.

Отличия

Основное отличие заключается именно в расположении полярности, что касается внешнего вида или даже технических характеристик, они могут быть абсолютно одинаковыми.

Но как бы там ни было, специалисты не советуют ставить аккумулятор, полярность которых не рассчитана под соответствующую модель. В случае нарушения полярности, электроника не только может отказаться работать, но и вовсе сломаться.

При выборе аккумулятора, важно ориентироваться не только на страну производителя, узнаваемость бренда и стоимость.

Что общего между этими аккумуляторами? — да практически всё!

Они оба функционируют согласно одним и тем же принципам. Среди отличий можно отметить следующие. Аккумуляторы прямой полярности в основном устанавливаются на азиатские и российские автомобили. Плюсовая клемма располагается слева. Аккумуляторы обратной полярности имеет плюсовую клемму справа и оснащается такими АКБ, чаще всего европейские и американские автомобили.

Какие будут последствия, если перепутал клеммы?

Последствия могут быть разными, исходя из того, в какой именно ситуации были перепутаны клеммы.

При установке на автомобиль

В том случае, если плюс «+» с минусом «-» на аккумуляторе были перепутаны в то время, когда АКБ устанавливался на автомобиль с работающим двигателем, тогда владелец транспортного средства получит достаточно большое количество неприятностей, начиная от того, что выйти из строя может диодный мост генератора, а также остальные электронные устройства в авто.

Чаще всего, такая ситуация случается со старыми транспортными средствами, где не было еще защиты от неправильного подключения АКБ.

Важно! Аккумулятор, который был подключен неправильно и надолго оставлен в транспортном средстве, может спровоцировать короткое замыкание и пожар.

В том случае, если клеммы будут перепутаны, а автомобиль в этот момент не будет заведен, тогда владельца ждут гораздо меньшие проблемы.

При такой ситуации, выйти из строя могут только приборы, которые были включены ранее, например магнитола, часы и т.д. Иногда, ситуацию спасают перегоревшие предохранители, которые монтируются в цепи питания, естественно, если они отвечают требованиям максимального тока в цепи.

 

Полезное видео

Вот пример последствий, которые произойдут в автомобиле, если перепутать полярность АКБ:

При зарядке аккумулятора

Во время зарядки аккумулятора, намного чаще путают клеммы местами, чем при его установке на автомобиль. Проблема заключается в том, что клеммы зарядных блоков «крокодилы» порой имеют одинаковый размер и внешний вид.

В том случае, если во время зарядки, было обнаружено, что полярность была перепутана, тогда просто необходимо изменить полярность и продолжить заряжать аккумулятор, предварительно проверив его на работоспособность.

В том случае, если ошибка была обнаружена уже после того, как аккумулятор был заряжен, тогда эта ситуация окажется немного сложней в разрешении. Это обусловлено тем, что внутри аккумулятора уже началась «переплюсовка». Говоря другими словами, теперь минус «-» стал плюсом «+» и наоборот.

Чтобы исправить эту ситуацию, для начала необходимо полностью разрядить свой аккумулятор. Делается это посредством включенных стоп-сигнала и габаритов. Также можно подключить автомобильную лампочку. Как только аккумулятор будет полностью разряжен, необходимо снова подключить его к зарядке, но в этот раз очень важно не перепутать клеммы и подключить его правильно. Как только зарядка будет окончена, снова можно пользоваться аккумулятором.

В том случае, если перепутать полярность аккумулятора во время домашней зарядки, то практически в 95% случаев, зарядное устройство выйдет из строя.

Практически все зарядки имеют специальный предохранитель, который в таких случаях сгорает, но сохраняет тем самым работоспособность аккумулятора. При такой ситуации, необходимо будет приобрести новый предохранитель и заменить его, а в следующий раз, внимательно следить за полярностью подключения клемм.

Заключение

Несмотря на то, что каких-то очень серьезных проблем неправильное подключение аккумулятора не принесет, иногда можно дорого поплатиться за сгоревшую электронику в автомобиле. В любом случае, очень важно внимательно смотреть на полярность и не торопиться при установке или зарядке аккумулятора.

Перепроверив лишний раз правильность подключения, можно обезопасить себя от лишних финансовых потерь, траты сил и времени. Если это случилось, не стоит впадать в панику, необходимо действовать согласно приведенной выше инструкции, тогда все будет хорошо.

Что будет если перепутать клеммы аккумулятора на машине

Содержание

  • Последствия при неправильном подключении клемм аккумулятора на двигателе
  • Неправильное подключение АКБ при зарядке
  • Если перепутать клеммы во время «прикуривания» автомобиля
  • Вопросы и ответы:

Большинство автовладельцев уверены, что аккумулятор – простое устройство и проблем с его использованием быть не должно. Единственная ошибка, поджидающая водителей – возможность перепутать клеммы при зарядке или установке АКБ на двигатель. В современных автомобилях плюсовая клемма больше размерами, поэтому даже при установке в полной темноте её легко можно обнаружить на ощупь.

Однако в неприятную ситуацию можно попасть при установке аккумулятора на ТС старого образца, а также при зарядке или прикуривании.

Подробнее тут: как правильно прикуривать автомобиль от другого автомобиля.

Зажимы-крокодилы одинаковы по размеру, поэтому легко подключаются к плюсу и минусу. Последствия нарушения полярности зависят от некоторых обстоятельств и модели автомобиля.

Последствия при неправильном подключении клемм аккумулятора на двигателе

Наиболее печальный вариант развития событий – запуск двигателя при неправильно подключённом аккумуляторе. Масштабы «катастрофы» зависят от скорости реакции водителя и модели авто. Могут возникнуть следующие проблемы:

  1. Замыкание. В 100% случаев запуск двигателя при неправильно установленном аккумуляторе чреват замыканием. В местах соединения появляются искры, слышатся щелчки и даже идёт дым. Дальнейшее развитие событий зависит от внимательности и скорости реакции водителя. При немедленном отключении зажигания и остановке двигателя можно обойтись «малой кровью»: оплавятся провода, а потом сгорит предохранитель. В этом случае достаточно заменить предохранитель и провода.
  2. Воспламенение. Игнорирование искрения приводит к пожару под капотом. Тонкие провода быстро оплавляются и воспламеняются. Учитывая близость бензина и масла, крайне велик риск пожара.
  3. Поломка ЭБУ. Выход из строя электроники не менее серьёзное последствие ошибки подключения. Оставшись без электронного «мозга», автомобиль попросту перестанет работать. Ремонт ЭБУ грозит автовладельцу серьёзными материальными затратами.
  4. Снижение мощности аккумулятора. Пластины АКБ при неправильном подключении вступят в процесс «переплюсовки» и начнут осыпаться. Следствие этого негативного процесса – падение мощности батареи.
  5. Выход из строя генератора. В лучшем случае сначала сгорит диодный мост, если он установлен на генераторе. Если его нет, перепутанная полярность закончится выгоранием генератора. На панели загорится лампочка аккумулятора. Это будет означать, что генератор необходимо заменить.

Неправильное подключение АКБ при зарядке

Вероятность неправильного подключения клемм во время зарядки АКБ гораздо выше. Чем при установке в двигатель, поскольку визуальной разницы у клемм «зарядников» нет. Развитие событий в этом случае может быть разным. В качественном зарядном устройстве сработает предохранитель, и процесс прервётся самостоятельно. Останется лишь сменить предохранитель и зарядить АКБ при правильном подключении. Использование дешёвого китайского зарядного устройства приведёт к полному его выходу из строя.

В некоторых случаях предохранитель не помогает, и зарядка продолжается. Если ошибка обнаружена своевременно, достаточно сменить полярность и продолжать процедуру зарядки.

В полностью заряженном аккумуляторе происходит внутренний процесс «переплюсовки». Подключать такой агрегат к двигателю, естественно, нельзя. Исправить ошибку можно, полностью разрядив батарею, подключив автолампочку или габариты. Как только АКБ полностью разрядится, её заряжают с соблюдением правильной полярности.

Если перепутать клеммы во время «прикуривания» автомобиля

Ошибка в подключении во время прикуривания – самый сложный случай, который может закончится неприятностями для обоих ТС. Каждый автомобиль испытает двойной удар: на проводку и на систему одновременно. Если прикуривание проводится при включённом двигателе, дополнительно пострадает генератор.

Несоблюдение полярности может привести к выходу из строя и даже взрыву АКБ меньшей мощности. Если не среагировать в течение 4-5 секунд, аккумулятору не хватит даже сил завести двигатель. Пострадать может также любой электроприбор: кондиционер, стеклоподъёмники, магнитола, сигнализация и т.п.

Последствия ошибок при подключении клемм в любом случае не сулят ничего хорошего. Даже секундная заминка может привести к выходу из строя нескольких узлов автомобиля, поэтому при подключении АКБ стоит быть предельно внимательным.

Вопросы и ответы:

В каком порядке подключать клеммы на аккумуляторе? Это зависит от особенностей установки аккумулятора. Главное при подключении плюсовой клеммы не замкнуть ее с подключенным минусом (не дотронуться до корпуса авто).

Что сначала подключать в аккумуляторе плюс или минус? Чтобы случайно не замкнуть электронику (закручивая гайку, можно дотронуться к кузову), при подключении клемм лучше сначала надеть плюсовую, а затем минусовую клемму.

Как правильно подключить зарядное устройство к аккумулятору? Вначале подключается плюсовая клемма, затем минусовая. Проверить прочность фиксации «крокодилов» (чтобы не искрило), затем включать ЗУ в розетку.

Как отключить аккумулятор в машине? Клеммы могут закиснуть, поэтому чтобы ключ не зацепил заземленный кузов, лучше вначале снять минусовую клемму, а затем крутить плюсовую. Затем откручивать крепеж АКБ.

Главная » Без Категории » Что будет если перепутать клеммы аккумулятора на машине

2022-05-26

By: Александр Фальченко

On:

In: Без Категории

Что будет если перепутать плюс с минусом

Содержание

Интересует вопрос: какие будут последствия, если перепутать клеммы аккумулятора? Мы рассмотрим данную тему, так как такую информацию должны знать начинающие автовладельцы. Трудно себе такое представить, что можно перепутать клеммы при установке аккумулятора.

При постановке АКБ на подзарядку это сделать можно, особенно в спешке. Сделать это на автомобиле гораздо труднее, ведь клеммы имеют разные размеры, но такое случается.

Так какие будут последствия, если перепутать и неправильно подключить клеммы аккумулятора? Чтобы дать правильный ответ на такой вопрос, необходимо рассмотреть возможные случаи такого подключения.

Начнем с самого легкого по последствиям случая, это когда перепутаны зажимы, которыми подключают зарядное устройство к аккумулятору. ЗУ не имеют клемм с разными размерами, у них они быстросъемные, и перепутать их легко. ЗУ заводского изготовления среагируют на это перегоранием предохранителя.

Самодельные ЗУ такой защиты могут не иметь, а знак об «аварии», они могут подать сильным гулом силового трансформатора. Если такая ошибка была быстро ликвидирована, то особых последствий для АКБ не будет.

Гораздо хуже для него, если он будет так «заряжаться» некоторое время. В таких случаях в АКБ происходит процесс, который специалисты называют переплюсовка. Она наносит вред аккумулятору, уменьшая его срок службы, но немного подправить ситуацию возможно. Для этого необходимо полностью разрядить АКБ с помощью автомобильной (лучше от стоп-сигнала) лампочкой. После этого уже правильно подсоединив ЗУ к аккумулятору, производят его полную зарядку.

Что произойдет, если перепутать?

Какие будут последствия, если перепутать клеммы аккумулятора на автомобиле?

Возможно несколько вариантов подключения и их последствий.

  • Клеммы перепутаны при установке на автомобиль при работающем двигателе;
  • АКБ установлен при выключенном зажигании.

Первый пункт доставит намного больше неприятностей водителю, чем второй. При смене полярности АКБ можно вывести из строя диодный мост генератора, а также другие электронные устройства автомобиля. Это касается в основном старых автомобилей, у которых не предусмотрена заводом защита от неправильного подключения АКБ. На большинстве современных генераторов установлены электронные реле, контролирующие зарядку аккумулятора, для которых смена полярности недопустима.

Меньшими последствиями обойдется неправильное подключение АКБ при выключенном зажигании. В таком случае обычно выходят ранее включенные электронные устройства, например магнитола, часы и другие приборы. Иногда выручают перегоревшие предохранители, установленные в цепи их питания, но при условии, что они соответствуют необходимым требованиям по максимальному току в защищаемой цепи.

Возможные неисправности

Оставленный надолго неправильно подключенный АКБ может вызвать пожар. Такие же последствия могут возникнуть при неправильном прикуривании от другого автомобиля.

Также может пострадать бортовой компьютер, если таков установлен на автомобиле. Это грозит полным отказом всех систем автомобиля. Необходима его замена, которая значительно «облегчит» кошелек владельцу.

Халатность и невнимание могут вывести из строя автосигнализацию. Она предпочитает работать только со своими полюсами.

Обязательно обратите внимание на проводку, причем не, только ту, которая идет от аккумулятора. Случается, что плавятся и замыкают провода, которые были под нагрузкой во время подключения. На современных авто предусмотрена некоторая защита от неправильного подключения. На плюсовых клеммах устанавливают предохранитель.

Мы рассмотрели вопрос: какие будут последствия если перепутать клеммы аккумулятора. Также для предотвращения замыкания электронные блоки защищают диодными мостиками с предохранителями. Предохранитель перегорает, а блок в исправном состоянии. Не проявляйте излишнюю поспешность при установке АКБ, она может дорого обойтись в прямом смысле.

Учимся на своих ошибках так сказать, а всему виной спешка и невнимательность.

Volkswagen Polo 1.9D 1998 года
тот самый что у меня в профиле. Как то раз я ставил на него АКБ, в спешке, а тут еще и одна из клемм поломанная, в результате чего минусовая могла одеться на плюсовую, при том что разного размера, и вот тот самый день когда я все же перепутал полярность и одел плюс на минус а минус на плюс.
При первом накидывании сразу начало искрить на клеммах и трещать клапана АБС, я быстро снял, и опять одел, не подозревая что всё причиной этому перепутанная полярность, и понял это только когда защелкало и заискрило второй раз. После я завел авто, значок АКБ горит, на клеммах всего 12 вольт, те что дает АКБ, то есть заряда нет.
Результат: сгорел генератор, предохранители целые

Honda Civic Hybrid 2008 года
в этот раз пострадал не мой авто, а авто клиента. случай был не стандартный, везли меня на чужой машине за город, там где стоял “пациент”, надо было снять зарядить АКБ и заменить ВББ, пока снимал ставил ВВБ, заряжался АКБ, тут пришло время его ставить, и как на зло АКБ нестандартного размера с большими клеммами, темновато, уже замерз, да и длинный язык, надо же им всё объяснить что как и почему.
В общем накидываю вторую клемму, и тут длинной паузой загорается аварийка, я быстро снимаю клемму и понимаю что перепутал полярность, после того как развернул АКБ, и накинул клеммы, ничего не загоралось, хотя должно было, в добавок при повороте ключа полная темнота.
Хорошо что тестер у меня с собой всегда, я сразу прозвонил все предохранители в трех коробках, и оказалось не прозванивается тот что самый большой, черный, двойной, на 100 и 70а, в добавок он еще на двух болтиках и не прозрачный.
Нашел отвертку у хозяйки, открутил болтики, достал его, открыл, а в нем перегоревшая нить, приличной толщины, и паяльника нет с собой, и проводов. и как на зло этого предохранителя небыло и у моего коллеги электрика, и вообще он дефицитный на разборе, а новый в эксисте 15уе и под заказ.
Результат: на сгоревший предохранитель делаем жучок от провода 220 и ищем за свой счет предохранитель.

Honda Accord 2008 2.4 АКПП
почти всё как на гибридном цывике, сгорел силовой предохранитель что в общей коробочке, его запаяли, завелась поехала дальше, но пропал звук, оказывается сгорел усилитель что стоит в багажнике под обшивкой правого крыла, ремонт усилителя обычно до 60уе, нам обошелся в 40.
как оказалось никакой защиты у этого усилителя нет, при этом есть предохранитель, но он не успевает перегореть, горит усилитель, да и вообще очень часто у аккорда эти усилителя горят сами по себе.

Volkswagen Passat B6 1.9 TDI
Всего сгорело около 4 предохранителей, где то в общей коробке предохранителей под капотом, в магнитоле, в на проводе что идет к генератору и где то еще, какие именно не уточнял, но вроде 4, генератор оказался исправный, проверяли на стенде, всё ок.

Для современных автомобилей эта проблема неактуальна – пространство под капотом настолько тесное, что и правильным-то образом клеммы на аккумулятор надевать не удобно. Однако человек на то и человек, чтобы иногда совершать невозможные поступки непреднамеренно. Можно купить сам аккумулятор несоответствующей полярности и поставить его на место старого. При прикуривании риск неправильно подключить клеммы аккумулятора есть у всех машин, в темноте или при спешке бывает и не такое.

Последствия неправильного подключения аккумулятора

Сразу развеем миф о том, что «задом наперед» подключенный аккумулятор запускает двигатель в обратную сторону. Начнем со стартера: если статор с обмоткой, а не с постоянными магнитами, то при переполюсовке направление тока изменится одновременно и в статоре, и в роторе, стартер начнет вращаться в ту же сторону, что и раньше. Вспомните о коллекторных двигателях электродрелей, болгарок и другого инструмента – они работают на переменном токе, направление которого меняется постоянно.

Стартер с постоянными магнитами на статоре действительно начнет вращаться в другом направлении, но двигатель провернуть не сможет. Бендикс не даст передать усилие в «неправильную» сторону, для того он и предназначен.

Даже если бы Вы и умудрились раскрутить мотор в обратном направлении, четырехтактный мотор при этом начинает забирать воздух из выпускного коллектора, а выбрасывать отработавшие газы во впускной. Так что ни карбюраторный, ни инжекторный двигатель так не завести, разве что дизель с механическим ТНВД запустится.

Если водитель перепутал полярность аккумулятора при установке, то одним из самых серьезных последствий будет то, что однозначно пострадает генератор. Посмотрите на схему его диодного моста (на примере простейшей «жигулевской» схемы):

Полупроводниковый диод – это элемент с односторонней проводимостью, не зря его символическое изображение похоже на стрелку. В направлении этой «стрелки» он проводит ток, в обратном – нет. Когда аккумулятор подключен правильно, то, пока генератор не работает (напряжение на его обмотках меньше, чем напряжение аккумулятора), диоды заперты – для аккумулятора генератор нагрузка, потребляющая несколько миллиампер из-за наличия в диодах небольшого обратного тока.

Если перепутать клеммы на аккумуляторе, то диоды в мосте откроются, и аккумулятор окажется замкнут через них. Это приведет к перегреву диодного моста и выходу диодов из строя с характерным запахом и хлопком из-за разрушения их корпусов.

Cгоревший диодный мост генератора

А вот для бортовой электроники переполюсовка вредна не всегда. Любое электронное устройство, если его не собирали на уроках труда китайские дети, имеет защиту от переполюсовки. Реализуется она просто с помощью тех же диодов: тут они открыты при «правильном» подключении, а при «неправильном» отрезают нагрузку от бортовой сети. Однако диоды по цепи питания редко используются в мощных потребителях тока – магнитолах и усилителях. А в них есть крайне уязвимые для переполюсовки элементы – электролитические конденсаторы, устанавливаемые для сглаживания пульсаций напряжения как раз в цепях питания.

Электролитические конденсаторы имеют высокую удельную емкость, поэтому популярны в электронике. Но их слабое место – это работа только при определенной полярности напряжения на обкладках, при переполюсовке происходит нагрев, обильное газовыделение (конденсатор вздувается), затем – взрыв. Чем больше емкость и габариты конденсатора, тем все это сильнее – если конденсатор на несколько микрофарад слегка щелкнет, то «банки» на тысячи микрофарад, применяемые в усилителях, взрываются.

Вышедшие из строя конденсаторы

И множество реле в проводке может издать тот же звук, что и диоды в мосте генератора. Дело в том, что в обмотках электромагнитных реле при отключении напряжения возникает резкий выброс напряжения, имеющего обратную полярность относительно источника питания. Для устранения вредного влияния таких импульсов на работу чувствительной электроники принято использовать диоды, подключаемые в обратном направлении. Пока на обмотку подано напряжение из бортовой сети, диод заперт, а при размыкании обмотки он открывается импульсом индуктивного выброса и гасит его. При переполюсовке аккумулятора такие диоды перегорят.

Короткие замыкания в сети

Что будет, если замкнуть клеммы аккумулятора? Учитывая способность стартерных аккумуляторов кратковременно отдавать ток в сотни ампер, предмет, замыкающий клеммы (например, случайно попавший между «плюсом» и кузовом автомобиля гаечный ключ), расплавится. Более массивные металлические предметы способны намертво привариться к клеммам или кузову, высаживая батарею «в ноль». В группе риска – автомобили, где аккумулятор установлен в багажнике – не стоит небрежно бросать туда ключи или буксировочные проушины, неизвестно, куда они попадут под действием тряски при езде.

Замыкание аккумулятора при попадание постороннего предмета

Поскольку подобные замыкания происходят скрытно от владельца (случайно упавший при ремонте ключ можно хотя бы сразу увидеть), то они наиболее опасны – высока вероятность воспламенения в багажнике.

При подобных замыканиях возможны проблемы с бортовой электроникой. Штатные предпусковые обогреватели на немецких автомобилях после такого блокируются, и их работоспособность восстанавливается только после принудительной разблокировки с диагностического сканера.

Автомобили с CAN-проводкой чувствительны к скачкам напряжения питания. Есть риск появления спорадических ошибок в памяти контроллеров.

Неправильное прикуривание

Если водитель перепутал клеммы аккумулятора при прикуривании, то происходит следующее: мы получаем две последовательно соединенных батареи, выходы которых замкнуты накоротко. Получаем короткое замыкание уже под напряжением в 24 вольта. Подавляющее большинство продающихся в магазинах комплектов проводов, у которых сечение жил и так недостаточно, при этом моментально сгорит, оставив на руках незадачливого водителя ожоги. Если же провода с достаточным сечением, то перед перегоранием они успеют нагреться.

Поэтому всегда проверяйте правильность подсоединения проводов. Если у заводских комплектов хотя бы есть цветовая маркировка, то самодельные провода чаще нарезаются из одного куска и по цветам не отличаются. Пометить их можно цветной изолентой на «крокодилах» — это снизит риск неправильного подключения.

Первыми соединяются «плюсовые» клеммы – сначала «крокодил» крепится на прикуриваемом аккумуляторе, затем на прикуривающем. Масса подключается в обратном направлении – сначала на прикуривающем аккумуляторе. Таким образом цепь замыкается на последнем шаге на прикуриваемом автомобиле.

Так как правильно отсоединять клеммы аккумулятора не менее важно, порядок отключения тоже строго определен. Первой отключается масса, после этого любые манипуляции с проводами не будут давать риск короткого замыкания.

Помните, что прикуривание для бортовой электроники – это нештатная ситуация, создающая повышенную нагрузку на генератор, чрезмерные скачки напряжения в сети прикуриваемого автомобиля во время вращения стартера.

Видео: Что будет если перепутать полярность на клеммах аккамулятора? Что произойдёт? Как устронить?

Что будет если перепутать клеммы аккумулятора на машине

Что будет если перепутать клеммы на аккумуляторе

Правильная эксплуатация транспортного средства, позволяет не только продлить его срок службы, но и предотвращает возникновение ситуаций, которые могут угрожать жизни водителя и пассажиров. Не все начинающие автолюбители знают, что будет, если перепутать клеммы аккумулятора на машине.

Данная ошибка, которая может показаться несущественной, часто становится причиной опасных последствий. Даже водители с большим опытом могут ошибочно поставить аккумуляторную батарею неправильно. Чаще всего такое происходит, когда источник тока имеет обратную полярность.

Что будет если перепутал клеммы при зарядке?

Если речь идет о зарядке дома через зарядное устройство, то нужно отметить что вашему аккумулятору скорее всего ничего не будет! В 95% случаев сгорит само устройство, ведь в нем стоит предохранитель, который от неправильного напряжения — просто сгорит, тем самым защитив и батарею, и сам «зарядник». Вам нужно будет поменять в нем предохранитель, после чего постараться правильно поставить провода для зарядки. У моего друга бывало такое, ничего страшного — после того как опомнился подсоединили провода к клеммам, ничего не происходило, то есть приборы на заряднике были по нулям и сила тока, и вольтаж. Однако если у вас дешевый Китайский зарядник, то он может полностью погореть, ничего не поделаешь нужно будет покупать новый!


Возможные последствия неправильной полярности

1: замыкание

Если поменять местами клеммы на АКБ, то сразу появятся электрические щелчки, и пойдут искры. Провода, на которых крепятся клеммные колодки, начнут быстро нагреваться. Если сразу их не снять с аккумуляторной батареи, то они могут оплавиться. Пойдет дым и едкий запах.

2: воспламенение, пожар

Оплавление проводов, на которых крепятся клеммы, может привести к их возгоранию. Пламя легко способно распространиться на другие детали проводки и автомобиля. Возле АКБ расположено много потенциально опасных элементов, которые быстро воспламеняются. Это может быть пластмасса, масло, бензин и пр.

Если своевременно не потушить источник возгорания, то автомобиль может полностью сгореть.

3: падение мощности АКБ

Внутри батареи находятся пластины с разной полярностью (т.е. плюсовые и минусовые). Они залиты электролитом. Пластины рассчитаны на то, чтобы каждая держала свой заряд.

Те элементы, которые должны держать минусовый заряд, стремятся стать плюсовыми. Та же ситуация происходит и с плюсовыми пластинами.

При длительной переполюсовке начинает происходить деформация пластин. Если часть из них разрушится, то аккумулятор потеряет свою мощность. Он будет быстро разряжаться. В зимний период его может даже не хватить на 1 завод авто. Если посыпятся пластины на старой аккумуляторной батарее, то прибор полностью выйдет из строя.

4: выход из строя электронного блока управления ЭБУ

Бортовой компьютер, электронный блок управления, мозги — это разные названия 1 прибора, который управляет всеми процессами в автомобиле. При замыкании клемм на АКБ можно спровоцировать его поломку. Если ЭБУ выйдет из строя, то автомобиль не сможет функционировать. Его нельзя будет завести, включить фары, дворники и пр.

Устранение данной поломки потребует больших финансовых затрат.

5: выгорание предохранителей

Такая поломка считается одной из самых минимальных. При перегорании предохранители берут весь удар на себя, т.е. другие системы автомобиля не страдают. После правильной установки АКБ следует проверить работоспособность всех функций. Выгоревшие предохранители потребуется заменить новыми.

6: поломка генератора

Чтобы защитить генератор от поломки, которая может произойти при неправильном подключении клемм, устанавливается диодный мост. Замыкание может привести к его сгоранию, но при этом оставляет дееспособным устройство, вырабатывающее электроэнергию.

Последствия при неправильном подключении клемм аккумулятора на двигателе

Наиболее печальный вариант развития событий – запуск двигателя при неправильно подключённом аккумуляторе. Масштабы «катастрофы» зависят от скорости реакции водителя и модели авто. Могут возникнуть следующие проблемы:

  1. Замыкание. В 100% случаев запуск двигателя при неправильно установленном аккумуляторе чреват замыканием. В местах соединения появляются искры, слышатся щелчки и даже идёт дым. Дальнейшее развитие событий зависит от внимательности и скорости реакции водителя. При немедленном отключении зажигания и остановке двигателя можно обойтись «малой кровью»: оплавятся провода, а потом сгорит предохранитель. В этом случае достаточно заменить предохранитель и провода.
  2. Воспламенение. Игнорирование искрения приводит к пожару под капотом. Тонкие провода быстро оплавляются и воспламеняются. Учитывая близость бензина и масла, крайне велик риск пожара.
  3. Поломка ЭБУ. Выход из строя электроники не менее серьёзное последствие ошибки подключения. Оставшись без электронного «мозга», автомобиль попросту перестанет работать. Ремонт ЭБУ грозит автовладельцу серьёзными материальными затратами.
  4. Снижение мощности аккумулятора. Пластины АКБ при неправильном подключении вступят в процесс «переплюсовки» и начнут осыпаться. Следствие этого негативного процесса – падение мощности батареи.
  5. Выход из строя генератора. В лучшем случае сначала сгорит диодный мост, если он установлен на генераторе. Если его нет, перепутанная полярность закончится выгоранием генератора. На панели загорится лампочка аккумулятора. Это будет означать, что генератор необходимо заменить.

Что будет, если перепутать клеммы и неправильно подключить аккумулятор?

Для современных автомобилей эта проблема неактуальна – пространство под капотом настолько тесное, что и правильным-то образом клеммы на аккумулятор надевать не удобно. Однако человек на то и человек, чтобы иногда совершать невозможные поступки непреднамеренно. Можно купить сам аккумулятор несоответствующей полярности и поставить его на место старого. При прикуривании риск неправильно подключить клеммы аккумулятора есть у всех машин, в темноте или при спешке бывает и не такое.

Что будет если перепутал клеммы при «прикуривании»?

Здесь последствия могут быть самыми плачевными для обоих автомобилей. Тут получается как бы двойной удар по системе авто и по проводам, с одной стороны ваш еще не совсем дохлый аккумулятор, с другой бодрый, да еще бывает многие и при заведенном двигателе подключают — а это означает что еще и генератор. Получается адская смесь — даже сложно представить что будет, но искры будут валить просто как от фейерверка, замыкание будет 100%, если не отсоедините провода в течении 1 — 2 минут начнут плавится и просто загорятся, а дальше как на этой фотке.

Если подвести итог, то получается — что последствия после такого «перепутывания» не очень хорошие, ребята если сами не понимаете в авто, лучше доверить дело профессионалам! И еще раз — внимание и только внимание, если перепутали, но вовремя «скинули», то возможно что ничего страшного не случится!

НА этом все, читайте наш АВТОБЛОГ.

(18 голосов, средний: 4,44 из 5)

Рекомендуем посмотреть:

Последствия неправильного подключения аккумулятора

Сразу развеем миф о том, что «задом наперед» подключенный аккумулятор запускает двигатель в обратную сторону. Начнем со стартера: если статор с обмоткой, а не с постоянными магнитами, то при переполюсовке направление тока изменится одновременно и в статоре, и в роторе, стартер начнет вращаться в ту же сторону, что и раньше. Вспомните о коллекторных двигателях электродрелей, болгарок и другого инструмента – они работают на переменном токе, направление которого меняется постоянно.

Стартер с постоянными магнитами на статоре действительно начнет вращаться в другом направлении, но двигатель провернуть не сможет. Бендикс не даст передать усилие в «неправильную» сторону, для того он и предназначен.

Даже если бы Вы и умудрились раскрутить мотор в обратном направлении, четырехтактный мотор при этом начинает забирать воздух из выпускного коллектора, а выбрасывать отработавшие газы во впускной. Так что ни карбюраторный, ни инжекторный двигатель так не завести, разве что дизель с механическим ТНВД запустится.

Если водитель перепутал полярность аккумулятора при установке, то одним из самых серьезных последствий будет то, что однозначно пострадает генератор. Посмотрите на схему его диодного моста (на примере простейшей «жигулевской» схемы):

Полупроводниковый диод – это элемент с односторонней проводимостью, не зря его символическое изображение похоже на стрелку. В направлении этой «стрелки» он проводит ток, в обратном – нет. Когда аккумулятор подключен правильно, то, пока генератор не работает (напряжение на его обмотках меньше, чем напряжение аккумулятора), диоды заперты – для аккумулятора генератор нагрузка, потребляющая несколько миллиампер из-за наличия в диодах небольшого обратного тока.

Короткие замыкания в сети

Что будет, если замкнуть клеммы аккумулятора? Учитывая способность стартерных аккумуляторов кратковременно отдавать ток в сотни ампер, предмет, замыкающий клеммы (например, случайно попавший между «плюсом» и кузовом автомобиля гаечный ключ), расплавится. Более массивные металлические предметы способны намертво привариться к клеммам или кузову, высаживая батарею «в ноль». В группе риска – автомобили, где аккумулятор установлен в багажнике – не стоит небрежно бросать туда ключи или буксировочные проушины, неизвестно, куда они попадут под действием тряски при езде.

Замыкание аккумулятора при попадание постороннего предмета

Поскольку подобные замыкания происходят скрытно от владельца (случайно упавший при ремонте ключ можно хотя бы сразу увидеть), то они наиболее опасны – высока вероятность воспламенения в багажнике.

При подобных замыканиях возможны проблемы с бортовой электроникой. Штатные предпусковые обогреватели на немецких автомобилях после такого блокируются, и их работоспособность восстанавливается только после принудительной разблокировки с диагностического сканера.

Автомобили с CAN-проводкой чувствительны к скачкам напряжения питания. Есть риск появления спорадических ошибок в памяти контроллеров.

Впервые озвучены цены на автомобиль от Xiaomi.

Но дебют китайской новинки состоится не скоро

Автоновости
Учитывая все вышесказанное нужно помнить, что если присутствуют сомнения в том, что автолюбитель в состоянии сам поставить аккумуляторную батарею правильно, то лучше не пожалеть денег и времени и обратиться к специалистам, так как последствия ошибки могут вылиться в дорогостоящий ремонт. Более того, эксперименты со своим автомобилем – это небезопасное мероприятие. Они могут повлечь за собой еще более негативные последствия, с которыми самостоятельно автолюбителю справиться уже не удастся. Автор: Олег Мокров

Неправильное прикуривание

Если водитель перепутал клеммы аккумулятора при прикуривании, то происходит следующее: мы получаем две последовательно соединенных батареи, выходы которых замкнуты накоротко. Получаем короткое замыкание уже под напряжением в 24 вольта. Подавляющее большинство продающихся в магазинах комплектов проводов, у которых сечение жил и так недостаточно, при этом моментально сгорит, оставив на руках незадачливого водителя ожоги. Если же провода с достаточным сечением, то перед перегоранием они успеют нагреться.

Отказ электронных блоков управления автомобиля

В случае переполюсовки клемм аккумулятора на общий провод автомобиля (массу) подается плюс от АКБ, на питающую шину следует минус. В этой ситуации все электронные блоки автомобиля запитываются в обратной полярности.

В большинстве автомобилей электронные блоки имеют двойную защиту на предмет превышения питающего напряжения более 15 Вольт и на переполюсовку. Обычно она организована при помощи специальных стабилитронов и предохранителей. Однако время срабатывания такой защиты может быть больше, чем время выхода из строя микропроцессорных устройств. В таком случае сначала выходит из строя, например, блок управления двигателем, далее – перегорают предохранители защиты. После этого требуется дорогостоящий ремонт блоков или их замена.

понятие и отличия, плюсовой провод аккумулятора автомобиля

Опытные автолюбители знают, чем чревато неправильно подключение аккумуляторных клемм. Причем полярность АКБ может отличаться в зависимости от производителя автомобиля, а также конструктивных особенностей транспортного средства. Этот материал позволит разобраться в вопросе полярности, а также возможных последствиях, которые могут произойти при неправильном подключении.

Содержание

  • 1 Что будет, если перепутать полярность
  • 2 Чем грозит владельцу авто спутывание полярности?
  • 3 Какие будут последствия, если перепутал клеммы?
    • 3.1 При установке на автомобиль
  • 4 Можно ли поменять полярность на аккумуляторе автомобиля
  • 5 Способы, как можно определить полярность
  • 6 Как правильно купить аккумулятор
  • 7 Прямая и обратная полярности
  • 8 Что значит прямая и обратная?

Что будет, если перепутать полярность

Многие неопытные водители после снятия аккумулятора на подзарядку устанавливают его неправильно, наоборот, путая плюс с минусом. При неправильном соединении проводов в контактах происходит сильное искрение, могут погореть любые элементы электрической схемы автомобиля. Но на современных машинах предусматривается защита, и в первую очередь при коротком замыкании выбивает предохранители.

Если вдруг водитель перепутал полярность аккумулятора, может произойти следующее:

  • перестанут показывать приборы на панели, погаснет приборная подсветка;
  • перегорит диодный мост или реле напряжения в генераторе;
  • выйти из строя блок управления двигателем;
  • сгорит силовой предохранитель большой мощности.

Чаще что-то выходит из строя в электрике японских автомобилей – азиатская техника не рассчитана на неправильное подключение.

Перепутать подсоединение аккумулятора нужно суметь, так как при таком подключении:

  • не хватает длины плюсового провода, его приходится сильно натягивать;
  • плюсовая клемма толстая, и на ней не фиксируется массовый провод.

Обычно при перепутывании полярности происходит кратковременное замыкание – вряд ли автовладелец сумеет надежно закрепить провода на клеммах. Искрение в момент контакта происходит очень сильное, к тому же клеммы в этом случает не подходят по размерам.

Чем грозит владельцу авто спутывание полярности?

Важно помнить: конструкция автомобиля предполагает использование только АКБ определенного типоразмера, расположения клемм. Установить в посадочное место аккумулятор аналогичной емкости, но не подходящей полярности возможно. Но вод подключить его в стандартном режиме будет уже невозможно. Так как провода просто не дотянутся.

Такая разница между АКБ присутствует не случайно: неправильная полюсовка станет причиной повреждения электроники автомобиля. Может выйти из строя блок управления двигателем, возникнуть КЗ. Важно помнить: не все электроприборы выходят из строя при смене полюсов. Но многие будут повреждены. Стоит избегать подобных ситуаций.

Причем в некоторых случаях отверстия в клеммах даже отличаются. Обычно диаметр положительной несколько больше, чем отрицательной. Потому даже случайно перепутать будет затруднительно.

Причем смена полюсов может стать причиной вращения стартера в другую сторону. Некоторые двигатели после такого могут потребовать капитального ремонта.

Какие будут последствия, если перепутал клеммы?

Последствия могут быть разными, исходя из того, в какой именно ситуации были перепутаны клеммы.

При установке на автомобиль

В том случае, если плюс «+» с минусом «-» на аккумуляторе были перепутаны в то время, когда АКБ устанавливался на автомобиль с работающим двигателем, тогда владелец транспортного средства получит достаточно большое количество неприятностей, начиная от того, что выйти из строя может диодный мост генератора, а также остальные электронные устройства в авто.

Чаще всего, такая ситуация случается со старыми транспортными средствами, где не было еще защиты от неправильного подключения АКБ.

Важно! Аккумулятор, который был подключен неправильно и надолго оставлен в транспортном средстве, может спровоцировать короткое замыкание и пожар.

В том случае, если клеммы будут перепутаны, а автомобиль в этот момент не будет заведен, тогда владельца ждут гораздо меньшие проблемы.

При такой ситуации, выйти из строя могут только приборы, которые были включены ранее, например магнитола, часы и т.д. Иногда, ситуацию спасают перегоревшие предохранители, которые монтируются в цепи питания, естественно, если они отвечают требованиям максимального тока в цепи.

Можно ли поменять полярность на аккумуляторе автомобиля

Сейчас большинство производителей стремятся исключить возможность неправильного подключения. Для этого минусовую и плюсовую клеммы делают разных размеров. Провода от бортовой сети имеют определенную длину и могут крепиться только с нужных сторон. Но владельцы автомобилей имеют возможность обойти защиту, подключив контакт через переходник.

Переходник на аккумулятор

В результате плюс становится не с той стороны, происходит переполюсовка, опасная для всех систем автомобиля. Конечно, некоторые автовладельцы специально переполюсовывают старый АКБ, чтобы продлить его ресурс.

Но эту операцию приходится производить на свой страх и риск, а последствия зачастую оказываются неприятными. Переполюсовка возможна только при следующих обстоятельствах:

  • Банки исправны и не замыкают.
  • Внутри батареи поддерживается нормальная плотность электролита.
  • Пластины достаточно толстые и не имеют повреждений.

Если срочно потребовалась замена аккумулятора, можно поставить прибор с несоответствующей полярностью. В сложившейся ситуации батарея должна быть установлена так, чтобы провод с положительным зарядом доставал до вывода. Минусовой — это масса, он присоединен к кузову, поэтому его можно удлинять. Для этого старый провод отсоединяют, а вместо него закрепляют новый, имеющий большее сечение. К сожалению, плюсовой не откручивается, его нельзя заменять и удлинять.

Способы, как можно определить полярность

Определить этот параметр для правильного подключения проводов можно путем внимательного осмотра устройства. Как сказано выше, для каждой полярности предусмотрена своя маркировка, а также символы возле клемм для подключения проводов. Более точно узнать этот параметр позволит сервисная книжка к автомобилю, а также советы, приведенные выше (автор видео — канал Ник86 авто-стройка).

В том случае, если на конструкции аккумулятора нет никаких отметок и признаков полярности, соответственно, вы не знаете, как подключить провода, то можно попробовать провести тест. Вам потребуется мультиметр, щупы которого нужно соединить с клеммами. В том случае, если напряжение на дисплее тестера будет положительное, это свидетельствует о том, что щупы были подключены верно (на самих щупах есть отметки «+» и «-»). Если же диагностика показало отрицательное напряжение, то прибор для тестирования вы подключили неверно.

Есть еще несколько способов определения:

  1. По цветовому обозначению. Маркировка цветом есть на многих современных батареях — в данном случае плюсовой вывод всегда отмечается красным. Так что спутать выводы не получится.
  2. По размерам штырей. Опытные автолюбители знают, что положительная клемма всегда больше по размерам, чем отрицательная. Такое отличие характерно для многих современных аккумуляторов и даже с износом устройства оно остается. Как вы понимаете, по таким способам определить параметр значительно проще, чем с мультиметром.

В батареях, выпущенных в США, штырей попросту нет — вместо них устройство оборудуется выемками, в которые монтируются токопроводящие контакты. 

Как правильно купить аккумулятор

Покупая аккумуляторную батарею в магазине или на авторынке, автовладельцу необходимо подбирать ее по основным параметрам:

  • размерам;
  • полярности;
  • емкости;
  • пусковому току.

Если АКБ будет слишком больших размеров, ее нельзя будет закрепить на аккумуляторной площадке, и нужно знать, что у незакрепленной аккумуляторной батареи из-за вибрации достаточно быстро осыпаются пластины. Полярность нового аккумулятора должна соответствовать типу автомобиля, и если водитель совсем не разбирается в машинах, необходимо хотя бы запомнить, как располагаются клеммы у старой батареи, или зарисовать, чтобы не забыть, расположение клеммных выводов. Еще важно выбирать такую же полярность, какая и была, чтобы не путаться – при неправильном подключении АКБ возникает короткое замыкание с неприятными для автовладельца последствиями.

Не стоит покупать АКБ уменьшенной емкости, для стандартного легкового авто без кондиционера и другого дополнительного электрооборудования аккумулятор должен быть не менее 55 Ампер-часов. Если в электрической схеме много потребителей, можно приобрести батарею и на 60 А·ч.

При покупке новой аккумуляторной батареи нужно стараться приобрести АКБ с максимальным пусковым током. Вреда двигателю от такой батареи не будет, зато стартер сможет прокрутить мотор в морозную погоду без проблем. Еще не будет лишним взять с собой вольтметр (мультиметр) и проверить напряжение на клеммах аккумулятора – у нормально заряженной батареи оно должно быть около 12, 5 Вольт.

Прямая и обратная полярности

Какой-либо разницы между АКБ с одинаковыми характеристиками, но разной полностью нет. Но размещение клемм в разных местах создает определенные проблемы, связанные с установкой. Существует устоявшийся перечень обозначений, позволяющий определить полярность. Определить данную характеристику можно в том числе и визуально. Порядок:

  • необходимо поставить АКБ стороной с клеммами ближе к себе, на горизонтальную поверхность;
  • расположение:
  • прямая полярность – плюсовая клемма слева;
  • обратная полярность – плюсовая клемма справа.

Все автомобили сконструированы под определенное расположение клемм. Прямая полярность обычно используется на автомобилях выпущенных на заводе АвтоВАЗ, а также машинах азиатского происхождения. Например, Kia, Daewoo, Hyundai и других. Прямая полярность разработана в советские годы отечественными конструкторами.

Именно поэтому многие продавцы в магазинах называют таковую полярность российской. Также прямую обозначают цифрой «1». Соответственно, на АКБ разных типов расположение клемм прямо противоположно. Обратный используется обычно на автомобилях зарубежного производства. Как правило, это машины марки Ford, большинство других американцев. Также большая часть производителей в Европе использует данный тип полярности.

Помимо обозначенных выше некоторые авто предполагают альтернативное расположение полярностей. «Американская» — предполагает, что клеммы для подключения к транспортному средств располагаются не сверху, а сбоку.

Что значит прямая и обратная?

Рассмотрим более подробно каждый вид аккумуляторов:

  • С прямой полярностью. Такая разработка актуальна в основном для отечественных инженеров. Ее особенность заключается в том, то вывод на плюс «+» располагается с левой стороны, а на минус «-» — с правой стороны верхней крышки корпуса.
  • С обратной полярностью. Такие батареи в основном применяются в странах Европы. Полярность располагается следующим образом: минус «-» находится слева, а плюс «+» — справа.

Справедливости ради, стоит отметить, что далеко не все автомобили из Европы имеют батареи с обратной полярностью, зачастую те модели, которые собираются в странах СНГ, комплектуются с прямой полярностью.

Обслуживанием АКБ многие автомобилисты занимаются самостоятельно, и одна из немногих возникающих проблем — определение полярности. Обычно узнать, где находится минусовая и плюсовая клеммы можно при визуальном осмотре. Если этого недостаточно, не потребуются дорогостоящие инструменты или оборудование. Нельзя эксплуатировать автомобиль, если нет полной уверенности, что аккумулятор подключен правильно. Когда самостоятельно проверку выполнить невозможно, стоить обращаться к специалистам.

Я случайно установил автомобильный аккумулятор задом наперед! Что мне с этим делать?

Мы не идеальные создания. Иногда мы делаем что-то неожиданно, например, неправильно устанавливаем автомобильный аккумулятор. Если вы не очень хорошо разбираетесь в установке батарей, вы можете подумать, что это не принесет никакого вреда. Но здесь мы имеем дело с автомобильными аккумуляторами, напряжение которых может искрить в любое время при неправильной установке.

Что делать, если вы устанавливаете автомобильный аккумулятор наоборот? Несчастные случаи могут произойти в любое время. Некоторые автовладельцы неправильно подключают свой массив. В этом случае транспортное средство или автомобиль не заведется. При обратном подключении автомобильного аккумулятора имеется предохранитель, который предназначен для защиты электроники автомобиля в случае ее перегорания.

Если в автомобиле нет этого защитного предохранителя, вам необходимо отправить электрический ток обратно в системы вашего автомобиля, включая блок управления двигателем, коробку передач и многое другое.

Когда ток течет обратно через фонари, это не будет проблемой. ЭБУ обычно не повреждается при обратной установке батареи. Большинство ЭБУ предназначены для работы против обратной полярности.

Неправильная установка батареи может привести к непосредственной опасности, например к искрению или электрическому удару. (Узнайте, как правильно установить автомобильный аккумулятор). Тем не менее, современные батареи уже рассчитаны на неправильную или обратную полярность. Производители аккумуляторов изменили это, потому что многие автовладельцы сталкивались с этим неправильным подключением аккумуляторов.

Таким образом, они разработали батареи, которые могут выдерживать обратную полярность или защищать пользователей от опасностей, связанных с подключением батареи с неправильной полярностью.

Шаги по поиску и устранению неисправностей автомобиля, который не заводится из-за неправильной установки аккумулятора

Шаг 1: См. предохранители с большим ампером на вашем автомобиле. В большинстве автомобилей используются большие предохранители, которые могут перегореть и предотвратить повреждение ECU/ECM. Это может быть предохранитель на 40, 60 или 80 ампер и это редкие находки.

При обратном подключении соединительных кабелей может перегореть предохранитель. После замены предохранителя можно заводить автомобиль. Он должен завести автомобиль, если не заводится, переходите к шагу 2.

Шаг 2: После того, как перегоревший предохранитель уже заменен, вы все равно можете столкнуться с проблемами при запуске двигателя автомобиля. Ваш двигатель может запуститься, но никогда не запустится. Когда это происходит, пришло время проверить все остальные маленькие предохранители. Вы должны убедиться, что предохранитель системы зажигания, ЭБУ, топливный насос и иммобилайзер все еще работают и находятся в хорошем состоянии.

Шаг 3: После проверки всего и всех предохранителей все еще в порядке, но ваш автомобиль все еще не работает, затем перейдите к этому шагу и сделайте следующее:

A. Машина заводится, но не заводится

● Проверьте, искрят ли свечи.
● Посмотрите на давление топлива.
● Проверьте, получают ли прямые форсунки достаточную мощность.

B. Автомобиль не заводится

● Проверьте систему иммобилайзера.
● Проверьте, получает ли ЭБУ достаточно энергии.
● Проверьте, подается ли питание на стартер.

Шаг 4: Если ничего не помогло, обратитесь к опытному механику для проверки вашего автомобиля. Эти шаги настоятельно рекомендуются всем, кто занимается самодиагностикой своих автомобилей. Вы также должны помнить, что нельзя допускать попадания на панель предохранителей дождевой воды или какой-либо другой жидкости.

Последствия неправильного подключения аккумулятора

Неправильное подключение аккумулятора может показаться простой проблемой, но подумайте еще раз, электрические повреждения, которые это может нанести вашему автомобилю и его аксессуарам, могут быть значительными. Вот некоторые последствия, которые могут возникнуть, если вы подключили автомобильный аккумулятор в обратном порядке.

● Генератор сгорит.
● Это повредит ваши радиоприемники и другие аксессуары.
● Это может привести к повреждению реле, компьютерного блока, замка зажигания или даже стартера
, если он включен.

Взорвется ли автомобильный аккумулятор, если подключить его наоборот?

Если вы имеете в виду подключение клемм аккумулятора в обратном положении или обратное подключение, то да, может взорваться. Вы можете повысить вероятность перегорания некоторых диодов и компонентов батареи при извлечении из батареи. Это также может означать, что громкого взрыва может не быть, но, возможно, это будет дорогостоящий результат.

Но под обратным вы подразумеваете, что ваша батарея подключена к массиву транспортного средства через пусковой толчок, и вы сделали это в обратном порядке, тогда, вероятно, может произойти настоящий взрыв.

Перемычки, расположенные в неправильном порядке, приведут к возникновению огромного тока, исходящего от обоих массивов. Однако, если взрыв не произошел внутри, то могла произойти мощная вспышка и плавящийся терминал от места последнего соединения. Поэтому, чтобы быть в безопасности, не пытайтесь это сделать.

Что произойдет, если вы неправильно заведете машину?

Все водители знают, что они могут оживить автомобильный аккумулятор, запустив его с помощью автомобиля с заряженным аккумулятором. Автомобильные аккумуляторы предназначены для обеспечения максимального электрического тока, необходимого для запуска двигателя. Еще более опасно, если соединительные кабели отсоединены.

Вот что может случиться:

1. Повреждение батареи

Если вы соедините положительную клемму с отрицательной клеммой батареи, это создаст сильный скачок тока на двух батареях. Он может быстро нагревать обе батареи, что для него не подходит. Тепло может расплавить все основные части клеток, как внутренние, так и внешние. Водородный газ может воспламениться и взорваться.

2. Повреждение соединительного кабеля

Кабели, используемые для запуска двигателя от внешнего источника, не должны подвергаться значительному скачку электрического тока. Он может быстро нагреваться до высоких температур. Из-за высокой температуры он может быстро расплавить оболочки кабелей и сделать кабели доступными для людей.

3. Неправильная полярность Повреждение

При неправильном подключении соединительных кабелей электрическая система автомобиля также будет изменена. Это может привести к непоправимому повреждению многих электронных компонентов автомобиля, таких как бортовые компьютеры и датчики.

Какие повреждения могут быть вызваны неправильно установленной батареей?

Ниже перечислены некоторые возможные повреждения батареи, установленной наоборот:

● Соединительные клеммы.
● Батарея.
● Зарядное устройство.
● Электрические компоненты автомобиля.

Итак, у вас есть следующие советы и приемы, как установить автомобильный аккумулятор задом наперед . Помните, что вы всегда должны практиковаться с осторожностью, особенно при работе с массивами.

Источники:
1. Неправильный запуск или установка аккумулятора – youcanic
2. Последствия подсоединения автомобильного аккумулятора задом наперед – Car Talk
3. Действительно ли автомобильные аккумуляторы взрываются, если их подключить наоборот? – Quora
4. Что произойдет, если соединительные кабели перевернуты на тесте? – Он все еще работает
5. Какие повреждения могут быть вызваны неправильным подключением кабелей зарядного устройства? – Techwalla

Почему батарея была повреждена, когда мы подключили провод батареи с неправильной полярностью?

    1. Спросите экспертов
    2. Образование

Вы должны войти в систему, чтобы опубликовать ответ.

  • Баллы: 3


    Возник вопрос о повреждении автомобильного аккумулятора при неправильном соединении проводов? Ищете причины этого повреждения? Здесь, на этой странице, наши специалисты ответили на ваш запрос.

    Поскольку мы знаем, что у автомобиля есть аккумулятор для запуска и два полюса, положительный или отрицательный, когда мы подключаем этот аккумулятор в неправильной полярности (положительный провод автомобиля к отрицательному полюсу аккумулятора или отрицательный провод автомобиля к положительному полюсу аккумулятора) в Последствия повреждения аккумулятора или перегорания автомобильной проводки могут привести к многочисленным потерям. Теперь я хочу знать, какая физика стоит за этим явлением, почему батарея может быть повреждена или часто теряется?

  • Ответы

    3 найденных ответа.

  • Баллы:

    4

    (₹ 4)

    Подключение аккумулятора в обратной полярности может привести к повреждению различных электронных и других компонентов автомобиля. Как правило, предохранители и диоды предназначены для защиты цепей в случае такой ошибки, но если они не выдерживают обратной полярности, как предполагалось, то вполне возможно, что в автомобильной электронике и других частях, таких как генераторы переменного тока и т. д., будет протекать высокий обратный ток. там будет много повреждений.
    На самом деле, когда мы неправильно подключаем аккумулятор, то вместо питания +12 вольт постоянного тока мы подаем -12 вольт в электросхему автомобиля, а это приведет только к разрушению, возгоранию и другим электрическим повреждениям. Если в вашем автомобиле установлены специальные предохранители и диодные защиты, то ущерб будет меньше. В старых автомобилях, как правило, придорожные механики имеют тенденцию ставить простой электрический провод вместо предохранителя в блоке предохранителей, и в таких случаях ущерб из-за неправильного подключения аккумулятора можно понять, поскольку этот провод не перегорает и не перегорает. не защищает цепь. В такой ситуации даже сама батарея повреждается из-за протекания через нее большого тока.


    Знание — сила.

  • Баллы:

    5

    (₹ 5)

    Известно, что в автомобилях обычно используются свинцово-кислотные аккумуляторы. Его также можно назвать аккумулятором. Аккумуляторная батарея — это устройство, которое преобразует химическую энергию в электрическую. Произведенная таким образом электрическая энергия будет использована позже, когда мы запустим автомобиль.

    Химические реакции, происходящие в этих вторичных клетках, обратимы. Эта обратная реакция будет иметь место, когда батарея подключена в обратном направлении. Когда ток изменится на противоположный, продукты, образовавшиеся в ячейке, вернутся к исходным материалам, которые сохранились. Когда батарея подключена в обратном порядке, будет иметь место обратная полярность, что приведет к следующим повреждениям.

    1. Все электронные компоненты могут сгореть.
    2. Это может привести к частичному или полному повреждению цепи зарядного устройства автомобиля.
    3, Аккумулятор может нагреться и взорваться.
    4. Аккумулятор может полностью разрядиться и выйти из строя.
    5. Тепло, выделяемое батареей с обратной полярностью, может привести к образованию газообразного водорода, который может взорвать корпус батареи. Это позволит кислоте из аккумулятора вытечь и повредить другие компоненты двигателя автомобиля.

    В наши дни появляются усовершенствованные версии аккумуляторов, в которых есть некий механизм защиты от этого обратного соединения, который будет использовать специальные диоды и предохранители. В таких случаях повреждений может не быть из-за перегорания предохранителей и отключения тока.

    Иногда, когда батарея полностью разряжается, выполняя это обратное подключение, некоторые люди пытаются зарядить батарею. Но это может не дать хороших результатов и не должно использоваться.


    DRRAO
    Всегда уверен

  • Точки:

    2

    (4. 2). сама батарея, электрические компоненты и даже вы можете навредить себе из-за вероятности серьезного взрыва в результате подключения неправильных клемм.
    Клеммы аккумулятора используют напряжение 12 В с каждой стороны клемм. Положительная сторона будет потреблять напряжение +12 В, в то время как отрицательная будет использовать -12 В. Если цепь будет работать, батарея попытается приспособить отрицательное 12 В к положительному, что вызовет избыточное накопление мощности. сопутствующее огромное количество тепла, которое выходит за пределы допустимого для системы. В конечном итоге это может привести к серьезному повреждению аккумулятора вплоть до взрыва.
    Повреждение автомобильного аккумулятора. Электролит в системе аккумулятора закипает из-за выделения избыточного тепла. Наконец, избыточное тепло может повредить внешнюю часть батареи, включая клеммы и пластиковую верхнюю часть, прикрепленную к батарее.
    Кроме того, это может привести к непоправимому повреждению генератора.


    Войдите, чтобы оставлять комментарии

    Отправить новую тему

    Вернуться к Вернуться к Спросить экспертов Раздел

    Клеммы автомобильного аккумулятора перекрещены. Что произойдет, если вы поместите кабели аккумуляторной батареи не с той стороны зарядка, а может быть, дело в том, что вы не так много ездите на своем транспортном средстве — это тоже может повлиять.

    В любом случае, если вы не можете завести машину, попробуйте запустить аккумулятор от внешнего источника. Это проверенное решение, которое вы можете рассмотреть.

    Запуск автомобиля от внешнего источника кажется легкой задачей, вы просто зажимаете разъемы и передаете заряд с исправного аккумулятора на разряженный.

    Но как бы просто это ни казалось, простая ошибка неправильного подключения клемм может означать конец для вашего автомобильного аккумулятора.

    Неспособность распознать, что является положительным, а какое отрицательным, не является обычным явлением, на самом деле, большинство людей действительно понимают это правильно. Но ради любопытства читателя давайте рассмотрим, что произошло бы, если бы клеммы поменялись местами.

    Содержание

    Риски, связанные с использованием соединительных кабелей в обратном направлении

    Автомобильные аккумуляторы немного сложнее, чем может показаться. При запуске автомобиля от внешнего источника есть определенные предостережения относительно зарядки вашего автомобиля, поскольку передача энергии от одной батареи к другой не так проста, как кажется.

    Неправильное подключение клемм аккумуляторной батареи может привести к серьезному повреждению самой аккумуляторной батареи, ее электрических компонентов и даже вам. Каждая клемма автомобильного аккумулятора использует ток 12 В с положительной и отрицательной ориентацией. Кабель на положительной клемме использует +12В, а на отрицательной стороне -12В.

    Если перепутать плюсы и минусы, батарея попытается компенсировать и превратить отрицательные 12 вольт в положительный заряд, что приведет к огромному скачку мощности и выделению огромного количества тепла. Излишне говорить, что это не будет хорошо переноситься системой или компонентами внутри и, в зависимости от определенных факторов, приведет к повреждению, которое в большинстве случаев будет серьезным и непоправимым.

    Повреждение кабелей-перемычек

    При мгновенном всплеске электричества первыми столкнутся с гневом и последствиями эксперимента кабели-перемычки. выделяющееся сильное тепло быстро расплавит изоляторы на соединительных кабелях, навсегда зажав их на батарее, которая будет следующей в очереди, поскольку пластиковая верхняя часть расплавится и согнется.

    Повреждение автомобильного аккумулятора

    Аккумулятор деформируется из-за того, что кислотные жидкости внутри него закипают из-за сильного тепла, выделяющегося в результате реакции. Кроме того, он может разлиться и повредить находящиеся поблизости детали, и вы, возможно, ничего не сможете с этим поделать.

    Перегоревший предохранитель/плавкая вставка

    Независимо от того, произошел ли у вас взрыв, в зависимости от других факторов и состояния аккумуляторов, вы, тем не менее, обязательно обнаружите в своем автомобиле перегоревшие предохранители и провода.

    Повреждение генератора

    Автомобиль, заряжающий неисправный аккумулятор, также может получить механическое повреждение. Всплеск мощности отрицательно скажется на генераторе и может даже нанести непоправимый вред.

    Физическое повреждение

    Если батарея находится в плохом состоянии или не использовалась некоторое время, запуск от внешнего источника с перепутанными кабелями может ухудшить ситуацию. Плохая батарея может даже взорваться и причинить вред окружающим.

    Что еще нужно учитывать при запуске от внешнего источника

    Перед запуском от внешнего источника ознакомьтесь с руководством по эксплуатации, чтобы узнать о мерах предосторожности или ноу-хау. В некоторых автомобилях вместо зажимов могут использоваться проушины, в то время как в некоторых автомобилях могут потребоваться определенные меры предосторожности, такие как удаление предохранителей или включение обогревателя. Некоторые новые автомобили не допускают запуска от внешнего источника и могут даже привести к аннулированию гарантии, поэтому обязательно ознакомьтесь с условиями гарантии.

    Не позволяйте двум автомобилям соприкасаться

    Выключите оба автомобиля, прежде чем найти аккумулятор

    Убедитесь, что клеммы не загрязнены, и если они есть, очистите их сухой тканью.

    Проверьте правильность напряжения с помощью мультиметра. Если напряжения не совпадают, это может привести к серьезным проблемам для обеих батарей.

    Снимите или выключите аксессуары и электрические устройства, такие как фары, радиоприемники и указатели поворота, так как скачок напряжения наверняка приведет к их короткому замыканию.

    Убедитесь, что автомобиль находится в нейтральном положении.

    Проверьте, не замерзли ли жидкости, не деформировалась ли батарея и не деформировалась ли она. Если да, то это может привести к взрыву.

    Всегда сначала начинайте с разряженной батареи, так как она не заряжена и безопасна в эксплуатации.

    Не оставляйте ничего под капотом при запуске автомобиля 

    После зарядки аккумулятора проедьте на автомобиле 15-20 минут, чтобы генератор полностью зарядил аккумулятор.

    Как правильно подключить соединительные кабели

    При потенциальном риске физического вреда и огромных затратах, связанных с заменой и перепрограммированием блоков управления двигателем, использование автомобильного аккумулятора для запуска двигателя без надлежащих знаний не является хорошей идеей и может привести к тяжелым последствиям. ремонт, если вы не будете осторожны с шагами.

    Автомобильный аккумулятор имеет две клеммы: положительную и отрицательную. Каждый терминал можно идентифицировать по его цвету и знаку, который он использует. Надежное соединение каждой клеммы чрезвычайно важно, так как слабое соединение может помешать процессу и не обеспечить эффективную зарядку аккумулятора.

    Теперь, когда мы разобрались с основами, давайте перейдем к очень важному вопросу, который требует от вас предельной осторожности и некоторой бдительности.

    В электронике положительная клемма всегда подключается первой, чтобы избежать встречи высокого напряжения полупроводников. При подключении соединительных кабелей всегда помните, что сначала нужно зажать кабели от положительной клеммы аккумулятора, которая обычно красного цвета и отмечена знаком плюс (+), и только после правильного подключения следует начинать подсоединять кабели от положительной клеммы аккумулятора. отрицательная клемма, обычно изображается черным цветом со знаком минус (-). Кроме того, следите за тем, чтобы какой-либо металлический предмет не касался одновременно обеих клемм аккумулятора.

    Чтобы лучше понять последствия отказа от подключения положительной клеммы, предположим, что сначала была подключена положительная клемма.

    Когда отрицательная клемма подключена, все шасси, включая металлические болты, удерживающие его на месте, заземляются, после чего установка положительной клеммы соединяет гаечный ключ с любой общей землей, что приводит к искрам, брызгам в лучшем случае, и дохлый аккумулятор или вообще взрыв блока в кошмарном.

    Рекомендуется сначала подключить положительную клемму, чтобы гарантировать, что гаечный ключ не сможет повредить корпус, даже если он коснется корпуса.

    Наоборот, при отсоединении кабелей рекомендуется начинать с отрицательной клеммы.

    Можно ли получить удар током при работе с автомобильным аккумулятором?

    Ток от системы 12 В постоянного тока недостаточно велик, чтобы причинить вам вред, на самом деле, возможно, даже не удастся обнаружить такие низкие напряжения, поскольку ток должен преодолеть сопротивление и пройти. Чтобы оказать на вас какое-либо воздействие, ток от батареи должен проникнуть под кожу, чтобы убить вас электрическим током.

    Высоковольтные источники тока сами по себе не представляют опасности. Однако есть несколько других вещей, на которые следует обратить внимание при работе с автомобильным аккумулятором, таких как утечка кислоты, искрение в свечах зажигания, соединительные кабели.

    • Фейсбук
    • Твиттер
    • LinkedIn
    • Копировать ссылку
    • Более

    Можно ли использовать батарею обратной полярности?

    Переполюсовка батареи – это случай, когда источник (для зарядки) или кабели нагрузки подключены неправильно. Это очень распространенная проблема. Но можно ли использовать батарею обратной полярности?

    Некоторые люди считают, что это обман, в то время как другие считают, что если это произойдет, батарея будет иметь ограниченную ценность. Потому что потоки тока теперь работают против того, как должны были работать внутренние плиты.

    В этой статье я собираюсь подробно обсудить обратную полярность батареи, причины этого, можно ли ее использовать и все остальное, что вам нужно знать.

    Что произойдет, если перепутать клеммы аккумулятора?

    Автомобильные аккумуляторы немного сложнее, чем кажутся. Существуют определенные ограничения на зарядку автомобиля при запуске от внешнего источника. Поскольку передача энергии от одной батареи к другой не так проста, как кажется.

    Перепутывание клемм аккумуляторной батареи может привести к серьезному повреждению аккумуляторной батареи, ее электрических компонентов и даже вам. Положительная и отрицательная клеммы автомобильного аккумулятора потребляют по 12 вольт тока.

    Положительная клемма кабеля питается от +12В, а отрицательная клемма питается от -12В.

    Если положительные и отрицательные стороны перепутаны местами, батарея попытается компенсировать это, преобразовав отрицательные 12 вольт в положительный заряд, что приведет к мощному всплеску мощности и большому количеству тепла.

    Излишне говорить, что это не будет легко переноситься системой или ее компонентами. И в зависимости от конкретных переменных это приведет к серьезному и необратимому ущербу.

    1. Повреждение соединительных кабелей

    Соединительные провода первыми пострадают от гнева и последствий эксперимента, если произойдет мгновенный прилив электричества.

    Выделенное сильное тепло немедленно расплавит изоляторы на соединительных кабелях, навсегда прикрепив их к батарее, которая будет следующей в очереди, поскольку пластиковая верхняя часть плавится и изгибается.

    2. Повреждение аккумулятора

    Кислотные жидкости внутри аккумулятора закипают из-за сильного тепла, создаваемого реакцией, что приводит к деформации аккумулятора. Кроме того, он может пролиться и повредить близлежащие детали, и вы не сможете предотвратить это.

    3. Перегоревший предохранитель

    Произойдет ли взрыв, зависит от других компонентов и состояния батарей. А вот перегоревшие предохранители и провода почти наверняка найдутся в вашем автомобиле.

    4. Повреждение генератора

    Автомобиль, заряжающий неисправный аккумулятор, также может получить механические повреждения. Увеличение мощности отрицательно скажется на генераторе, что может привести к необратимому повреждению.

    5. Физический урон

    Пуск от внешнего источника с перевернутыми кабелями может привести к ухудшению состояния неисправной батареи или батареи, которая не использовалась в течение длительного времени. Неисправный аккумулятор может даже взорваться, причинив травмы находящимся рядом людям.

    Может ли обратная полярность повредить электронику?

    Можно ли использовать батарею обратной полярности, не повреждая электронику? Вот в чем дело: обратная полярность может быстро повредить ваши гаджеты при подключении к розетке с неправильным подключением.

    Это ускоряется, когда нет встроенного механизма защиты от обратной полярности. Обратная полярность также может повредить или вызвать короткое замыкание ваших устройств, таких как телевизоры, радиоприемники, телефоны и ноутбуки.

    Поскольку электронное оборудование регулирует напряжение только одним способом, при обратном электрическом потоке внутренние компоненты могут перегреться или вызвать короткое замыкание.

    Будет ли GFCI работать с обратной полярностью?

    GFCI не нужно знать, какой провод заземлен, потому что ток между двумя проводами разный. Поскольку кнопка тестирования не зависит от EGC, как подключаемый тестер, полярность не должна быть проблемой.

    сообщите об этом объявлении

    GFCI, который не сбрасывается с обратной полярностью, должен использовать EGC.

    Можно ли использовать батарею обратной полярности?

    Аккумулятор будет полностью разряжен. Вы могли бы потенциально зарядить его отрицательно и продолжать использовать, но ваши пластины сделаны из двуокиси свинца с положительной стороны и губчатого свинца с отрицательной стороны.

    Теперь все будет наоборот. Перевернутая батарея будет работать только в ограниченной степени, потому что она больше не отформатирована правильно.

    Правда в том, что свинцово-кислотная батарея не может изменить свою полярность без внешней стимуляции.

    Как исправить обратную полярность батареи?

    Если батарея имеет обратную полярность по ошибке, следующий метод поможет исправить это.

    • Полностью разрядите аккумулятор – для этого следует подключить маломощную лампочку без схемы отключения.
    • Правильно подключите зарядное устройство. Если аккумулятор отказывается заряжаться, попробуйте на несколько секунд использовать более сильное зарядное устройство (например, 24-вольтовое зарядное устройство для 12-вольтового аккумулятора), прежде чем переключиться на правильное зарядное устройство с минимальными настройками.

    Примечание: Глубокие разряды повреждают внутренние компоненты, поэтому ожидаемый срок службы батареи при любых обстоятельствах сокращается.

    Заведется ли автомобиль, если аккумулятор перевернут назад?

    Автомобиль не заведется, если аккумулятор перевернуть. Если аккумулятор подсоединен наоборот, сгорит предохранитель, предназначенный для защиты электронных компонентов.

    Если в автомобиле нет предохранителя, предназначенного для этой функции (почти во всех автомобилях он есть), электрический ток будет течь в обратном направлении через системы автомобиля, включая ЭБУ, блок управления коробкой передач и другие компоненты.

    Когда ток течет обратно через диоды в электронных устройствах, таких как ECU/ECM (блок/модуль управления двигателем), это не проблема.

    Отсоединение проводов аккумуляторной батареи не является частым способом повреждения блока/модуля управления двигателем.

    Подавляющее большинство из них рассчитано на обратную полярность. Если диод выходит из строя, ECU/ECM можно снять и проверить в худшем случае.

    Как проверить обратную полярность с помощью мультиметра?

    Чтобы проверить обратную полярность с помощью мультиметра, выполните следующие действия:

    1. Вставьте черный провод в средний вход в нижней части мультиметра. Затем вы можете подключить красный провод к правому входу. Теперь вы можете включить мультиметр, повернув ручку посередине в положение «Напряжение переменного тока».
    1. Один провод можно поместить в более длинное вертикальное отверстие розетки. Это нулевой нейтральный контакт. Вы также можете разместить другой провод на более коротком вертикальном отверстии, которое представляет собой горячий контакт на 120 вольт.

    Если мультиметр показывает показания, значит полярность правильная, поскольку напряжение передается от горячего контакта к нейтральному.

    1. Поместите один провод в отверстие нейтрали, а другой провод в отверстие заземления. Если ваш мультиметр не показывает показания, обратной популярности нет.

    Таким образом, напряжение перемещается между контактами нейтрали и земли. Оба имеют нулевое напряжение.

    Почему обратная полярность опасна?

    Нейтральный и горячий провода в электрической розетке или цепи должны быть подключены к правильным клеммам. Если вы этого не сделаете, вы будете удивлены изменением полярности. Это может даже в конечном итоге представлять угрозу для вашего дома или автомобиля.

    Пока у вас обратная полярность, ток течет через вашу розетку, даже когда ваши приборы выключены. Между тем, даже при отсутствии тока могут произойти казусы, если вы по ошибке прикоснетесь к поврежденному или неправильному участку гаджета.

    Электрические розетки с обратной полярностью могут стать причиной возгорания, поражения электрическим током и короткого замыкания. В противоположной полярности опасен даже простой торшер.

    Обратная полярность также имеет тот недостаток, что может повредить бытовую технику и другие электронные гаджеты.

    Если к неподходящей части цепи подается напряжение, устройство останется активированным, даже если оно было выключено.

    Кроме того, существует значительный риск поражения электрическим током и перегрева.

    Часто задаваемые вопросы (FAQ):

    Что произойдет, если зарядить аккумулятор с неправильной полярностью?

    Батарея может взорваться, если зарядить батарею наоборот.

    сообщите об этом объявлении

    Имеет ли значение полярность при зарядке аккумулятора??

    Да, это абсолютно важно.

    Как исправить обратную полярность?

    Переключив провода на соответствующие стороны.

    Заключительные слова

    Если вы дочитали до этой части статьи, то уже знаете, можно ли использовать батарею обратной полярности или нет.

    Если вы хотите спокойствия и более комфортной жизни, рекомендуется проверить мультиметром обратную полярность. Это сэкономит вам деньги в будущем, предотвратив чрезмерные расходы, вызванные обратной полярностью.

    Похожие сообщения:

    • Как узнать, разряжена ли батарея AAA?
    • Можно ли заряжать аккумуляторы любым зарядным устройством?
    • Можно ли перезаряжать аккумуляторы DeWalt?
    • Можно ли использовать аккумулятор DeWalt на 60 В с инструментом на 20 В?
    • Можно ли использовать аккумулятор Flexvolt с инструментом на 20 В?

    Автомобиль с перепутанной полярностью, внутренняя электроника не.

    .. Автомобиль с обратной полярностью, внутренняя электроника не работает…

    Задайте вопрос, получите ответ как можно скорее!

    ×

    ЗАПРОСИТЬ ЦЕНУ

    спросил

    Спенсер Т

    на 26 марта 2017 г.

    Я спрыгивал с машины своей подруги и случайно поменял полярность на ее конце (приемном конце), когда они были подключены. Звуковые сигналы, дворники и датчики начали сходить с ума. После этого машина вообще не заводилась даже при правильном подключении. Отнес аккумулятор в передовые автозапчасти, там его зарядили. Собрал обратно, теперь машина заводится и работает. Но салонное радио, плафон, электрические стеклоподъемники не переключаются с парковки, а кондиционер не работает. Я поменял предохранитель на 80а, который был перегоревшим в двигателе, и это не помогло. Проверил все остальные предохранители, все не перегорели. Кто-то упомянул, что это может быть реле, но я не думаю, что оно запустилось бы, если бы это было так. Спасибо за помощь.

    Пробег моей машины 195000 миль.
    В моей машине установлена ​​автоматическая коробка передач.

    Электрические компоненты не работают Осмотр $94,99 – $114,99 Получить предложение

    Кевин Гейнер

    Автомеханик

    35-летний опыт микропроцессоры на транспортном средстве, например, в модуле управления трансмиссией (PCM). Что касается других предохранителей, то их множество, не говоря уже о плавких вставках, автоматических выключателях и всех отдельных электронных компонентах во многих цепях, которые могли выйти из строя из-за обратной полярности. Что касается цепей, которые вы упомянули, а именно радио, плафон, электрические стеклоподъемники, блокировка парковки и кондиционер, лучше всего просто проследить эти цепи, чтобы найти неисправность. Это может быть плавкая вставка, неисправность устройства или повреждение микропроцессорного управления или твердотельного реле. Простая трассировка цепи, например, просто начав с цепи плафона, выявит неисправность. Если вы хотите, чтобы эти шаги выполнял сертифицированный механик, отправленный YourMechanic прямо к вам, пожалуйста, запросите электрическую диагностику, и ответивший сертифицированный механик решит эту проблему для вас. Если у вас есть дополнительные вопросы или проблемы, не стесняйтесь снова обращаться к YourMechanic, поскольку мы всегда здесь, чтобы помочь вам.

    Заявления, приведенные выше, предназначены только для информационных целей и должны быть проверены независимо. Пожалуйста, смотрите наш условия обслуживания подробнее

    Получите мгновенную смету для вашего автомобиля

    К вам приедут наши сертифицированные механики ・Гарантия на 12 месяцев и пробег 12 000 миль・Справедливые и прозрачные цены

    Узнать цену


    Механик со стажем?

    Зарабатывайте до $70/час

    Подать заявку

    Что спрашивают другие

    Утечка масла в течение двух недель

    Двигатель может подтекать маслом на ходовую часть, где вы этого не видите, но более вероятно, что двигатель сжигает масло. Обычно это происходит при износе направляющих клапанов или поршневых колец. Если их слишком много…

    Honda N WGN 660 5cc japanese частый хромой домашний режим

    Здравствуйте, спасибо, что написали о своей Honda N WGN. Вероятная возможность, связанная с пропуском зажигания, может заключаться в том, что компьютер автомобиля мог попытаться перекалибровать подачу топлива в результате неисправного реле управления дроссельной заслонкой…

    Двигатель снова заработал после замены свечей зажигания и проводов.

    Здравствуйте. Если физических симптомов нет, вероятно, потребуется повторное сканирование компьютера и получение кодов неисправностей (https://www.yourmechanic.com/article/how-to-read-and-understand-check-engine-light- коды Джейсона Унрау). Индикатор Check Engine (https://www.yourmechanic.com/services/check-engine-light-is-on-inspection) может буквально сигнализировать о любой из сотен возможных проблем. Коды будут на…

    Замена передних тормозных колодок и приемлемая дилерская стоимость

    Привет. Я понимаю вашу озабоченность. Я начинаю предлагать замену тормозных колодок на 3 мм. У большинства техников есть измеритель толщины тормозных колодок, который избавляет от догадок при определении того, когда следует предложить замену тормозных колодок; 1-3мм красный, 4-6мм…

    Моя машина не заводится

    Здравствуйте. Если двигатель заведется и не запустится, нам нужно будет проверить, не хватает ли ему искры, топлива или компрессии. Если искры нет, то необходимо диагностировать систему зажигания. Если бы…

    лампочка “проверьте двигатель” и автомобиль дрожит

    Привет! Изучив первый код ошибки, который вы опубликовали, выяснилось, что проблемы с вибрацией и тряской на вашем VW, скорее всего, вызваны утечкой вакуума. Код P2279 OBD-II (https://www.yourmechanic. com/article/p2279-obd-ii-trouble-code-intake-air-system-leak-by-jay-safford) указывает на то, что ECU обнаружил неисправность. вакуум…

    Индикатор выключения трассы и индикатор двигателя загорелись одновременно, когда я включил двигатель.

    Привет. Индикатор Trac Off предназначен для контроля тяги. Компьютер отключил противобуксовочную систему по разным причинам. Возможно, неисправен контроллер, датчик скорости автомобиля или датчики скорости вращения колес…

    Я слышу, как вращаются колеса, когда замедляюсь.

    Привет, Камеша. При пробеге в 45 000 миль и с предоставленной вами информацией я сначала подумал, что звук может исходить от тормозных колодок. Незнание того, какой звук издает автомобиль, делает его довольно сложным…

    2001 Ford Taurus Проблемы с запуском

    Это может быть признаком неисправной катушки зажигания. Катушки зажигания (https://www. yourmechanic.com/services/ignition-coil-replacement) покрыты лакоподобной изоляцией, которая со временем становится хрупкой после многократного растяжения, а затем сжатия. В изоляции образуются небольшие трещины, которые открываются при нагревании и…

    Статьи по Теме

    Как проверить напряжение автомобильного аккумулятора

    Мало что может разочаровать больше, чем обнаружить, что ваш автомобиль не заводится из-за разрядки аккумулятора. Механик может измерить…

    Признаки неисправного или неисправного генератора

    Общие признаки включают необходимость частого запуска автомобиля от внешнего источника, тусклое освещение при управлении автомобилем или загорание индикатора аккумулятора.

    Общие сведения об аккумуляторах электромобилей

    Знайте, как работает аккумулятор вашего электромобиля, чтобы максимально увеличить срок его службы и эффективность, и покрывается ли замена гарантией производителя.


    Просмотрите другой контент

    Оценки

    Техническое обслуживание

    Города


    Аккумулятор обратной полярности — зарядное устройство, крепление и защита — информация об аккумуляторе

    • Лучший литиевый аккумулятор 18650

    • Цилиндрическая литий-ионная батарея

    • Руководство по лучшим литий-ионным батареям

    • Руководство по лучшим батареям LiPo

    • Лучшее руководство по батареям Lifepo4

    • Направляющая для литиевой батареи 12 В

    • Литий-ионный аккумулятор 48 В

    • Соединение литиевых батарей параллельно и последовательно

    • Лучший литий-ионный аккумулятор 26650

    29 марта 2022 г.   Просмотр страницы:995

    Обратная полярность — это когда отрицательная и положительная полярность батареи перепутаны. Когда к устройству подключается батарея с обратной полярностью, установленная на заводе вилка с перепутанными изготовителем проводами предотвратит неправильное ее подключение. Это также известно как концепция полностью разряженной вторичной батареи и неправильного подключения зарядного устройства к клеммам, чтобы сделать положительную клемму отрицательной, а отрицательную – положительной.

    3.2V 20A Низкотемпературная аккумуляторная батарея LiFePO4 -40℃ 3C разрядная емкость≥70% Температура зарядки: -20~45℃ Температура разрядки: -40~+55℃ пройти тест на иглоукалывание -40 ℃ максимальная скорость разряда: 3C

    ПОДРОБНЕЕ

    Зарядное устройство с обратной полярностью

    Во время зарядки батареи кабели могут быть случайно перепутаны и они могут быть подключены к неправильным клеммам – условие, которое называется обратной полярностью. Переполюсовка происходит, когда положительный кабель соединяется с отрицательным, и наоборот. Когда это происходит, аккумулятор, а также различные другие связанные с ним электрические компоненты могут быть повреждены.

    При неправильном подключении кабелей и клемм может возникнуть обратная полярность. Ток течет в неправильном направлении, когда происходит изменение полярности. В таких ситуациях, если кто-либо прикоснется к устройству, оно может быть повреждено, а человек может получить удар током.

    Как обратная полярность влияет на батарею?

    Последствия неправильной полярности включают в себя:

    Повреждение батареи – когда к клеммам случайно подключены неправильные кабели, полярность батареи может измениться, и батарея может разрядиться. Также при полной разрядке автомобильного аккумулятора его можно считать пустым сосудом. На таком этапе может быть переполюсовка автомобильного аккумулятора при подключении кабелей не к тем клеммам. В некоторых случаях батареи оставались в таком состоянии годами.

    Но это может быть опасно. Если процесс с обратной полярностью приводит к дополнительному выделению тепла, батарея может излучать дополнительное тепло. В редких случаях это может привести к взрыву батареи. Это может привести к выбросу расплавленного пластика и кислоты из аккумулятора, что чревато серьезными травмами. Это необходимо предотвратить любой ценой.

    Повреждение зарядного устройства – Неправильное подключение кабелей влияет на работу систем зарядного устройства и аккумулятора. Если есть изменение полярности клемм, неправильная полярность может быть отправлена ​​​​обратно в зарядное устройство. В таких случаях зарядное устройство может быть повреждено безвозвратно. Однако в некоторых случаях возможно частичное повреждение зарядного устройства. В таких случаях плата будет взиматься постепенно.

    Низкотемпературная высокая плотность энергии Прочный полимерный аккумулятор для ноутбука Спецификация батареи: 11,1 В 7800 мАч -40℃ 0.2C пропускная способность ≥80% Пыленепроницаемость, устойчивость к падению, защита от коррозии, защита от электромагнитных помех

    ПОДРОБНЕЕ

    Обратная полярность и электрические компоненты. к обратной полярности.

    Генератор переменного тока может быть компонентом, который может быть подвержен наибольшему риску повреждения. Его замена может обойтись довольно дорого. Предохранители в автомобиле могут перегореть, если через другие компоненты пройдет обратный ток.

    Если это не так, замена ЭБУ может оказаться дорогостоящей. Однако на самом деле навигационная система и развлекательная система могут стоить дороже. Также может случиться так, что корпус компьютера и различные другие компоненты также могут быть повреждены. Любой двигатель или часы (аналоговые) могут ненадолго вернуться назад, прежде чем они будут окончательно повреждены. Но большинство передовых автомобилей сегодня имеют защиту от обратной полярности на своих электронных модулях. Таким образом, кроме, возможно, перегоревшего предохранителя, ничего не произойдет.

    Обратная полярность Батарея Fix

    В случае обратной полярности, будет проводимость разработанного диода, и блок питания будет закорочен так, что появится заземление и сработает предохранитель, который перегорит. Это может защитить ваше оборудование. Если оборудование было правильно спроектировано, оно должно хорошо работать.

    Обратную полярность батареи можно изменить только несколькими способами. Лучшее решение — полностью разрядить аккумулятор. Ключ нужно либо оставить включенным, либо заряд полностью рассеется за несколько дней из-за незаметного короткого замыкания.

    Ваши электронные устройства могут быть легко повреждены из-за обратной полярности при неправильном подключении к розетке или проводке. Если в вашей электронике нет встроенного механизма, обеспечивающего защиту от обратной полярности в схеме, в любой момент может возникнуть риск повреждения.

    Защита батареи от обратной полярности

    Функции защиты могут быть электронными или механическими. Примером механической защиты является необходимость использования обучающих символов и изображений, а также специальных разъемов. Например, есть разные терминалы в 9-В батарея. Блоки перезаряжаемых батарей физически предназначены для вставки с одного направления.

    Другие типы батарей, например одноэлементные щелочные, не так легко защищаются механическими средствами защиты. Поэтому производители и разработчики оборудования с батарейным питанием должны убедиться, что любое обратное напряжение смещения и обратный ток достаточно низки для предотвращения повреждения внутренней электроники оборудования или самой батареи.

    Как правило, чтобы предложить такие электронные средства защиты, производители выбирают либо транзистор, либо диод для защиты батареи от переполюсовки. Используя диод, биполярный транзистор или полевой МОП-транзистор, можно построить схему защиты батареи от переполюсовки.

    Во время некоторых несоответствующих действий, случайного короткого замыкания или неправильной установки батареи, повреждения внутренней электроники, некоторые электронные меры безопасности – это то, что ожидают пользователи. Как правило, целесообразно использовать схему защиты батареи перед внутренней схемой или после батареи любого электронного устройства, чтобы иметь возможность защитить срок службы батареи, а также само электронное устройство.

    Добавить комментарий

    Ваш адрес email не будет опубликован. Обязательные поля помечены *